Made Up Questions

You might also like

Download as pdf or txt
Download as pdf or txt
You are on page 1of 113

1. A 29-year-old man is rushed to the emergency department unresponsive.

He was shot in the


back of the head with a pistol in a gang fight 20 minutes ago. Physical examination shows
adducted arms with flexion at the elbow. The knees are extended and the feet are plantar
flexed at the ankle. This patient is most likely suffering from a lesion of which part of the
central nervous system (CNS)?

A) Vermis of the cerebellum


B) Midbrain above the red nucleus
C) Midbrain at the red nucleus
D) Caudal pons below the locus coeruleus
E) Medullary pyramids of the medulla oblongata

2. A 72-year-old woman is brought to the physician by her 29-year-old son after an episode of
uncontrollable spasm. The son said that his mother has recently been having difficulty
speaking and controlling her own body movements. After the diagnosis of the mother is
made, the son decides to get genetic testing for himself. It is revealed that he has an
expanded CAG trinucleotide repeat. What is likely to happen to the son in the next several
years?

A) He is likely to remain healthy because his father is healthy.


B) He is likely to develop similar symptoms at age 72.
C) He is likely to develop similar symptoms at age 45 due to anticipation.
D) His fate is unpredictable.

3. A five-year-old boy is brought into the clinic by his mother because of newly erupting rashes.
His temperature is 37.4 ​°C ​(99.4 ​°F). The patient’s mother says that he has been very tired
recently and has rhinitis in the mornings. Physical examination shows fresh rashes adjacent
to crusting lesions that are in the healing process. Treatment is started with oral acyclovir.
What is the most likely outcome for this patient after treatment?

A) The infection is completely eradicated.


B) The infection lies latent in the trigeminal ganglion.
C) The infection lies latent in the sacral nerve ganglia.
D) The infection lies latent in the dorsal root ganglia.
E) There will be a Jarisch-Herxheimer reaction.

4. An 11-year-old boy is brought to the clinic by his father because of difficulty in weight
management. The father says that his son has recently been homeschooled because of
difficulty in keeping up with classes. When asked, the boy says homeschooling is a relief
from being made fun of for being in special education. Physical examination shows a boy at
50th percentile in height and 97th percentile in weight. Poor muscle tone is noted.
Cytogenetic studies reveal no abnormalities. What is the most likely cause of this patient’s
condition?
A) Deletion in maternal chromosome 15
B) Deletion in paternal chromosome 15
C) Methylation of maternal chromosome 15
D) Methylation of paternal chromosome 15
E) Increased stress from school workload.

5. A 24-year-old woman visits her local dermatologist because of a hyperpigmented lesion on


her right arm. She says that she has been using over-the-counter hydroquinone to try to
remove the discoloration, but instead the area is getting larger. Physical examination shows
a somewhat oval lesion 1 cm long, 0.6 cm wide, with irregular borders. A biopsy is taken and
malignant melanoma is confirmed. Upon learning this discovery, the patient becomes very
irritable. She asks if the physicians good online ratings were written by family members. She
says the slightly longer than average wait time is a sign of incompetence and demands to
see a different physician. What defense mechanism is this patient employing?

A) Denial
B) Displacement
C) Acting out
D) Blocking
E) Projection

6. A couple discusses with a genetic counselor about having a child. The woman is blood type
B and the man is blood type AB. Assuming that alleles B and O are of equal frequency in the
general population, what is the probability that they will have a child with blood type AB?

A) 0%
B) 12.5%
C) 25%
D) 37.5%
E) 50%

7. A 5-month-old child is brought to the emergency room by his mother sub responsive. She
says that her baby crawled into the garage and was accidently locked in for 15 minutes
while the car engine was running. Physical examination shows mildly blue skin. APGAR test
is 5, and respirations are 22/min (normal 30-60). A few minutes later the child vomits. What
additional finding is likely to be discovered in this child?

A) Decreased hemoglobin affinity for oxygen


B) Increased hemoglobin affinity for oxygen
C) Poor muscle tone
D) Retinal bleeds
E) Hearing loss
8. A 56-year-old man comes to the physician complaining of a spinning sensation for the past
four hours. His temperature is 37.2 °C (99.0 °F) and blood pressure is 190/92 mm Hg. A CT
scan of the brain stem shows a small hemorrhagic infarction above the pontomedullary
junction on the left. Rupture of the anterior inferior cerebellar artery (AICA) is suspected.
What neurologic related dysfunction is expected on physical examination of this patient?

A) Nystagmus to the left


B) Nystagmus to the right
C) Hearing loss in the left ear
D) Hearing loss in the right ear
E) Weakness of facial expression

9. A 28-year-old male computer analyst rushes into the urgent care clinic complaining that he
thinks he has diabetes mellitus. He insists that his dry skin and recent blurry vision are
symptoms of the disease. He says his grandma has diabetes. When he became a registered
bone marrow donor, he received his HLA profile which shows he is DR3 positive. Physical
examination shows 170 cm (5’7 in) in height and 97.5 kg (215 lbs) in weight with a BMI of
33.7. Blood glucose level is 104 mg/dL (normal 70-100). What should the physician's next
response be to this patient?

A) When was your last meal?


B) How often do you exercise?
C) Your BMI is high. Let’s work to reduce it.
D) Please stay in the clinic for a complete lab workup.
E) May I look at your complete HLA profile?

10. In a science experiment, an unknown bacteria protein is injected into a mouse. The protein
is engulfed by a host cell which then presents a piece of the protein to a helper T cell
through the use of its MHC II molecule. The naive T cell ​differentiates into​ T helper 2 cells.
What is the identity of the host cell?

A) Classic macrophage
B) Alternative macrophage
C) B lymphocyte
D) Mast cell
E) Neutrophil

11. Chester Southam was an oncologist who worked at Memorial Sloan Kettering Cancer
Center in the 1960’s. He conducted an experiment on the transmissibility of cancer using
two different groups of participants. Group one consisted of immunocompromised patients,
and group two consisted of healthy prison inmates. Both groups were injected with identical
clones of cancer. What was the result?

A) Both groups perished


B) Group one perished, group two became immune
C) Group one became immune, group two perished
D) Both groups became immune
E) The results were inconclusive

12. A 34-year-old woman gravida 4, para 1, abortus 3, gives birth to a full term healthy baby girl
with no apparent abnormalities. The mother had undergone cytogenetic testing in the past to
help explain her previous three spontanious abortions. The mother’s karyotype is shown.

​https://mymds.bham.ac.uk/genetics/reciptrans.htm

What meiotic event most likely led to the normal development of the child?

A) Alternate segregation
B) Adjacent-1 segregation
C) Adjacent-2 segregation
D) Monosomy rescue
E) Trisomy rescue

13. A 37-year-old woman visits her physician for a routine physical. She eats healthy and
exercises regularly. Occasionally she smokes a cigarette on the weekends during social
events to ease stress from work. Physical examination shows no abnormalities. She tells the
physician that she has recently decided to register with a bone marrow donor program
because she wants to save a life. Her HLA profile is shown.

What disease is this patient most likely to develop?

A) Rheumatoid arthritis
B) Multiple sclerosis
C) Systemic lupus erythematosus
D) Celiac disease
E) Graves disease

14. A researcher is studying somatic cells in mice. He notices a group of cells interact with
thymic epithelial cells and develop into Th​0​ cells. Th​0​ then travels to a peripheral lymph node
where it interacts with an MHC II molecule on a macrophage. Another group of cells in the
thymus develop into cells that secrete interleukin-10 (IL-10) and transforming growth factor-β
(TGF-β). What do all mentioned cells have in common?

A) They are lymphocytes


B) They are phagocytes
C) They are naïve cells
D) They are CD4+
E) They are antigen presenting cells

15. A mosquito bites a human and obtains merozoites. The merozoites undergo sexual
reproduction in the mosquito's stomach wall. The newly formed sporozoites are transferred
into a different human where it replicates via mitosis in the hepatocytes. What type of host is
the human?

A) Primary host
B) Definitive host
C) Intermediate host
D) Alternate host
E) Paratenic host

16. A physician in Alaska is visited by two biological sisters. The younger sister has naturally
lighter skin than her older sister. The younger sister goes tanning in Mexico for a week. In a
subsequent visit, both sisters are of equal dark complexion. At this point in time how is the
younger sister’s skin different from her older sister’s skin?

A) There is no difference because both sisters have equal skin tone.


B) The younger sister has more melanocytes.
C) The younger sister has smaller melanosomes with increased transport.
D) The younger sister has larger melanosomes with decreased transport.
E) The younger sister has increased skin shedding.

17. What accounts for the differences in natural skin color?

A) Number of melanocytes
B) Size of melanocytes
C) Size of melanosomes
D) Transport rate of melanosomes
18. What effect does sun exposure have on melanocytes?

A) Increase melanocyte numbers


B) Increase melanocyte size
C) Increase melanosome size
D) Increase melanosome transport

19. A 31-year-old woman comes to the clinic for advice on family planning. She says she is
taller than her father and brother. Physical examination measures 186 cm (6’1 ft). She has
wide set eyes and long slender legs. Histology from a cheek swab shows two Barr bodies in
the nucleus. What is this patient’s diagnosis?

A) Turner syndrome (45,X)


B) Metafemale syndrome (47,XXX)
C) Klinefelter syndrome (47,XXY)
D) Jacobs syndrome (47,XYY)

20. A 28-year-old man visits his dermatologist because of hyperpigmented spots on his arm. He
says it takes forever for the discoloration to disappear every time he picks at a bug bite. His
arm is shown.

What is the most likely reason for the long lasting hyperpigmented spots on this patient?

A) Continued scratching
B) Proliferation of melanocytes
C) Melanin in the dermis
D) Melanin in the epidermis
E) Chronic inflammation

21. ​A mother has hemophilia A and the father has hemophilia B. What is the chance that their
daughter will have hemophilia?

A) 0%
B) 25%
C) 50%
D) 75%
E) 100%

22. A 17-year-old boy comes to the physician because of newly erupting rashes on his face.
Physical examination shows red rashes on the forehead and cheeks. White pimples are
scattered throughout the rashes. Pus oozes from the pimples when popped. Tretinoin is
prescribed. What is the most likely causal organism?

A) ​Staphylococcus aureus
B) ​Streptococcus pyogenes
C) ​Propionibacterium acnes
D) ​Moraxella catarrhalis
E) ​Rickettsia rickettsii
F) ​Chlamydia trachomatis
G) ​Treponema pallidum

23. A 55-year-old man visits his family physician because of a chronic cough and profuse
sweating while asleep. He is a chicken farmer near Cleveland, Ohio. During the past year he
has lost 5.4 kg (12 Ibs). A chest x-ray shows calcifying lesions. Histologic examination of his
sputum shows small yeasts inside macrophages. What is the most likely causal organism?

A) ​Cryptococcus neoformans
B) ​Clamydophila psittaci
C) ​Mycobacterium tuberculosis
D) ​Coccidioides immitis
E) ​Blastomyces dermatitidis
F) ​Histoplasma capsulatum

24. A mother is a known carrier of hemophilia. She is concerned that her male child in utero has
the disease. A marker is identified 10 centimorgans (cM) away from the recessive
(diseased) hemophilia allele. The other X chromosome does not have this marker. An
amniocentesis is performed. The X chromosome of the male fetus is probed using
fluorescence in situ hybridization (FISH) and it is discovered that he has the marker. What is
the chance that this patient has hemophilia?

A) 100%
B) 90%
C) 80%
D) 70%
E) 60%

25. A scientist discovers a snap cap tube in the laboratory with the label “killed virus.” How
would she determine if the virus contains RNA or DNA?
A) Base sequence
B) Single or double strandedness.
C) 2 prime hydroxyl group
D) Length of the molecule

26. A 77-year-old man comes to the physician because of difficulty walking for the past three
months. Physical examination shows bradykinesia, rigidity, and hand tremors. A drug is
prescribed. The patient’s muscles relax and tremors disappear. However, bradykinesia
persists. This drug exerts which of the following actions?

A) Decrease dopamine degradation through inhibition of catechol-O-methyltransferase


B) Increase dopamine release into the synaptic cleft
C) Increase dopamine levels through an exogenous precursor
D) Upregulate dopamine receptors in the basal ganglia
E) Blocks acetylcholine receptors in the basal ganglia

27. A 66-year-old man presents to the physician with shortness of breath. He has gained 12 kg
(26 Ibs) in the past month. Chest X-ray shows an enlarged heart with an ejection fraction of
36%. Sacubitril and valsartan are started. What is the purpose of valsartan?

A) To reverse the inhibition of renin caused by sacubitril


B) To compensate for the lost activity of neprilysin
C) To counteract the toxic side effects of sacubitril
D) To induce quick excretion of sacubitril through activation of the P450 system
E) To increase sacubitril’s antagonist actions against neprilysin

28. A seven-year-old boy is brought to the physician by his mother because of shortness of
breath and chills. The boy seems noticeably uncomfortable. His temperature is 38.3 ​°C (101
°​F) and pulse is 95/min. Physical examination shows several pus filled lesions on the
cheeks. The mother states that her brothers and her eldest son also had similar conditions
at the patient’s age. Culture of a fluid filled pus grows yellow colonies on mannitol salt agar.
What is the mechanism of immune evasion of the most likely causative organism?

A) Breakdown of self-made hydrogen peroxide


B) Creation of fibrin clots to protect against phagocytosis
C) Production of toxic shock syndrome toxin-1
D) Production of cytolysins
E) Cleavage of host IgA antibodies

29. A 32-year-old man is brought to the emergency room unresponsive. His brother found him in
their backyard lake 20 minutes after he went fishing out on his boat. Despite appropriate
care being given, the patient slips into an indefinite coma. The patient has no proxy or
advance directive. His wife says that he has told her several times that he never wants to be
on life support. She says the patient would want his life support discontinued. The parents of
the patient say that he would say no such thing and that it would be extremely
uncharacteristic of their son to bring up a specific issue like life support on a ventilator. They
tell the physician to keep the patient intubated. The brother is unsure. What should the
physician do?

A) End life support because his wife is next of kin


B) Continue life support because the two parents hold majority opinion
C) Wait for the brother’s opinion because he found the patient
D) The physician should make his own best judgement given the situation.
E) Call in the ethics committee

30. A 49-year-old man is brought to the emergency room 20 minutes after being found
unresponsive in a nearby lake. His temperature is 36.7 °C (98.1 °F), heart rate is 71 bpm,
and blood pressure 140/90 mm Hg. A cold caloric test shows no twitches. The pupils do not
respond to light. Two separate neurologists confirm these findings. A toxicology test shows
up negative. The family states that they want the patient to be kept intubated. They say it
would be his wish to be kept alive. What should the physician do?

A) Keep the patient intubated because it is the families wish


B) Keep the patient intubated because it is the patients wish
C) Remove the tube and end all life support
D) Consult her colleagues on the best next step
E) Call in the ethics committee

31. A 22-year-old man is brought to the emergency room unresponsive 20 minutes after being
pulled out from a lake. Despite life saving measures, he slips into an indefinite coma. Which
of the following physiologic abnormalities is expected to be observed in this patient?

A) Hyperthermia
B) Hypertension
C) Tachycardia
D) Hyporeflexia

32. A 45-year-old woman comes to the physician complaining of unusual fatigue for the past few
weeks. She claims that she has lost 5.4 kg (12 Ibs) in the past month unintentionally. Her
blood pressure is 104/60 mm Hg and respirations are 14/min. Physical examination shows a
thin tan woman. Her lab values are shown below.

Sodium: 100 mEq/L


Potassium: 5.6 mEq/L
Hemoglobin: 12.5 g/dL
Hematocrit: 37%
Blood pH: 7.3
Mean corpuscular volume: 85 fL

Metyrapone administration shows no increase in plasma 11-deoxycortisol levels. What is the


most likely underlying physiology of this patient’s condition?

A. Autoimmune disease
B. Malignancy
C. Infection
D. Vitamin deficiency
E. Iron deficiency
F. Trauma

33. A 57-year-old woman visits her primary care physician for a routine physical. She has no
complaints. Her blood pressure is 169/109 mm Hg and respirations are 15 breaths/minute.
Her blood pressure was 118/80 mm Hg six months ago. Lab results are shown.

Sodium: 150 mEq/L


Potassium: 3.3 mEq/L
Hemoglobin: 16 g/dL
Hematocrit: 40%
Blood pH: 7.49
Mean corpuscular volume (MCV): 91 fL

A tap of the tragus of the facial muscle stimulates a twitch of the nose and mouth. This
patient most likely has what underlying condition?

A. Acute renal failure


B. Congestive heart failure
C. Inferior vena cava constriction
D. Adrenal adenoma
E. Hepatic cirrhosis

34. A couple comes to the clinic for genetic counseling. The woman is pregnant and she is
concerned her child might have phenylketonuria (PKU). They have a family history of PKU.
A family pedigree is shown.

What is the probability that their son (arrowhead) has PKU?


A. 1/12
B. 1/8
C. 1/6
D. 1/4
E. 1/2

35. Two researchers theorize that environmental pollution increases the risk of lung cancer.
They decide to test out this hypothesis by creating two groups of participants. One group
lives in New York City and the other group lives in rural Cameron County, Pennsylvania. The
groups are to be followed for a period of 10 years. What critical rule of this study must be
obeyed?

A. The two groups must have the same number of participants.


B. All selected participants must be disease free.
C. Sex, age, and ethnicity must be the same in both groups.
D. The study must be double blinded.
E. The participants must not have any family history of lung cancer.

36. A 24-year-old woman is rushed to the emergency clinic by a friend after she collapsed 15
minutes ago. The friend says that she took seven shots of vodka in celebration of
completing her first marathon run. The patient appears to be dazed and oriented to the
person, but not place or time. What is the most likely cause of this patient’s syncope?

A. Hypotension
B. Alcohol intoxication
C. Delirium tremens
D. Hypoglycemia
E. Dehydration

37. A 19-year-old man is brought to a physiatric clinic for behavior problems. He frequently runs
red traffic lights and does not have a license. Immunofluorescent imaging shows that he fails
to recognize green numbers within a red background. His family pedigree is shown.

What is the chance that this patient’s nephew (arrowhead) has the same condition?
A. 0%
B. 25%
C. 33%
D. 50%
E. 100%

38. A 15-year-old boy visits his physician due to recent muscle cramps and fatigue when playing
on his high school football team. He says that when he is tackled he has to use his hands to
“walk up” to a standing position. Physical examination shows slightly enlarged calves.
Laboratory results indicate elevated serum creatine kinase. A pedigree of the patient’s
family is shown.

His sister is pregnant. What is the probability that this patient’s sister’s child (arrowhead) will
have the same condition?

A. 0
B. ⅛
C. ⅙
D. ¼
E. ½

39. A 16-year-old biker comes to the emergency room after getting mauled by a couple stray
dogs on his way home from school. He hops on his left leg, although his right foot remains
close to the ground. When asked to flex his right knee, he says he is unable to do it.
Physical examination shows multiple bite marks around the right thigh. What are the most
likely origins of the damaged nerve?

A. L2-L3
B. L3-L4
C. L4-L5
D. L5-S1
E. S1-S2

40. A 26-year-old woman comes to her physician complaining of blistering sores on the edges
of her mouth. She says they come and go, but she is extremely annoyed that they always
appear when she is stressed. Oral acyclovir is prescribed. Two weeks later, she returns to
the physician complaining that the medication is useless. What is the most likely cause of
the ineffectiveness of acyclovir for this patient?

A. The drug was blocked by “first pass” metabolism in the liver.


B. The drug’s half life was too short.
C. The drug was consumed by macrophages.
D. The virus lacks thymidine kinase (TK).
E. Acyclovir only works as a topical medication for this type of infection.

41. A 20-year-old man is rushed to the emergency room with extreme nausea, vomiting and
diaphoresis. He was bitten by a rattlesnake an hour ago. His blood pressure is 100/60 and
respirations are 32 breaths/minute. Physical examination shows an ​erythematous rash with
two puncture wounds on the dorsum of the left wrist. Antivenin is quickly started. This
antivenin is cultured by inoculating rattlesnake venom into a horse and then retrieving the
horse’s serum. Administration of this antivenin into the patient is an example of what type of
immunity?

A. Natural active
B. Natural passive
C. Artificial active
D. Artificial passive

42. A 49-year-old woman visits her physician because of vision problems. She works the night
shift as a security guard at an oil mining outpost. Recently she says the day seems to get
darker faster and everything seems a lot more dim around her at work. Her past medical
history is significant for hyperthyroidism treated with radioactive iodine. Physical examination
shows yellow skin. Pupils are equal, round, and reactive to light. What is the most likely
cause of this patient’s symptoms?

A. Liver cirrhosis
B. Pancreatic tumor of the head
C. Excessive breakdown of heme
D. Decreased absorption of ​β-carotene
E. Decreased conversion of ​β-carotene to vitamin A

43. A young healthy couple arrives at the clinic for genetic counseling because the man has a
family history of miscarriages. A cytogenetic analysis of the man’s karyotype shows a
translocation between chromosome 4 and 6 [46,XY,t(4;6)(q21;q14)]. He does not have any
physical or mental abnormalities. Karyotype analysis of the woman shows no abnormalities.
If the couple were to have a child, what are the chances the baby will be phenotypically
normal?

A. 20%
B. 25%
C. 33%
D. 50%
E. 75%

44. A 15-week-old boy is brought to the physician by his mother because of cough, fever, chills,
and shortness of breath. Physical examination shows white plaques on the tongue that can
be scraped away. A chest x-ray shows pneumonitis. Lab testing reveals low serum IgG. An
inquiry into family history uncovers the following pedigree.

The physician tells the mother that the incidence of this disease is 1/50,000. How many
people in the general population are carriers of this disease?

A. 1/1,000
B. 1/20,000
C. 1/25,000
D. 1/50,000
E. 1/100,000

45. A 56-year-old man with chronic alcoholism visits his physician for help with his addiction. He
has been taking naltrexone for the past four months as instructed. But he feels that instead
of naltrexone helping him, he has been building up a tolerance to it. Now he is reverting
back to his old habits and would like a stronger medication. Disulfiram is prescribed. What is
the mechanism of action of this new drug?

A. Alcohol dehydrogenase agonist


B. Alcohol dehydrogenase antagonist
C. Acetaldehyde dehydrogenase agonist
D. Acetaldehyde dehydrogenase antagonist
E. Increase excretion of acetic acid

46. A 31-year-old woman with chronic alcoholism returns to her physician for help. She has
been taking naltrexone for the past three months as instructed, but she feels that instead of
it helping her, she has been building up a tolerance to it. Now she is reverting back to heavy
drinking. The physician prescribes disulfiram. What is a side effect of this new drug?
A. Nausea and vomiting
B. Decreased sexual ability
C. Neck pain
D. heachache
E. Shortness of breath

47. A 37-year-old woman visits her physician because of unusual physical changes. She is a
jogger and notes that her sneaker size has been steadily increasing. Her winter gloves that
she has worn for years now seem a little tight. She also notices moderate weight gain
despite being active. Physical examination otherwise reveals no obvious abnormalities.
Serum lab results are shown.

Sodium: 139 mmol/L


Potassium: 4.0 mmol/L
Glucose: 123 mg/dL
Insulin-like growth factor-1 (IGF-1): 1327 ng/mL
Cortisol: 10 ug/dL

Oral glucose is given and no apparent changes in IGF-1 are seen. This patient most likely
has which of the following underlying conditions?

A. Pre-diabetes mellitus
B. Atrophy of pancreatic β-islets
C. Pituitary macroadenoma
D. Pituitary adenocarcinoma
E. Ectopic releasing hormone secretion

48. A 14-year-old boy is brought to the physician by his father because of vision problems. He is
a quarterback at his local high school but has been increasingly tackled down from left and
right without seeing his attackers. He says his vision started to decrease in the lower corners
but has creeped upwards. He has grown 5 cm (2 inches) in the past year. Physical
examination shows the pupils are equal, round, and reactive to light. Goldman visual field
test reveals bilateral hemianopia. Serum lab results are shown.

Sodium: 142 mmol/L


Potassium: 3.9 mmol/L
Glucose: 71 mg/dL
Cortisol: 2.3 ug/dL
Growth hormone (GH): 30 ng/mL
Insulin-like growth factor-1 (IGF-1): 327 ng/mL

What is the most likely diagnosis?

A. Retinoblastoma
B. Medial and lateral rectus palsy
C. Pituitary adenoma
D. Pituitary adenocarcinoma
E. Craniopharyngioma

49. A 28-year-old woman was brought to the physician by a friend because of paranoia and
hallucinations for the past 3 days. The friend says that for the past two years the patient has
lost her affection for animals and no longer speaks with sophistication. She is prescribed an
antipsychotic. One month later the patient comes back to the physician by herself
complaining of impaired vision and a feeling of dizziness. An electrocardiogram (ECG) is
performed and the result is shown.

​Courtesy of Jer5150

Which medication was most likely prescribed to this patient?

A. Chlorpromazine
B. Thioridazine
C. Fluphenazine
D. Haloperidol
E. Amitriptyline

50. A 64-year-old man visits his physician because of fatigue and chronic dry coughs that have
persisted for a month. He is having increasing difficulty climbing the stairs to his apartment
due to shortness of breath. He admits to being a moderate smoker at social gatherings, but
smokes no more than one or two cigarettes a week. Physical examination shows clubbing of
the fingers. A pulmonary function test shows the following graph.
What is the most likely diagnosis?

A. Emphysema
B. Bronchitis
C. Bronchiectasis
D. Idiopathic pulmonary fibrosis (IPF)
E. Asthma

51. A 12-year-old girl is brought to the pediatrician by her father for a well child exam. She is
cheerful and has no complaints. Blood pressure measurements with a ​sphygmomanometer
reveals increased pressure in the upper limbs and decreased pressure in the lower limbs.
She is at the 13th percentile for height and 65th percentile for weight. Physical examination
shows low set ears and a webbed neck. Which of the following abnormalities is most likely
present in this patient?

A. Severe intellectual disability


B. Hypothyroidism
C. Horseshoe kidney
D. Bicuspid aortic valve
E. Coarctation of the aorta

52. A 34-year-old man visits his physician because of a heachache that is not relieved by
acetaminophen. He returned home from a summer camping trip two days ago where he
swam in a forest lake. He is prescribed metaxalone and sent home. The following day he is
rushed to the emergency room after his wife found him collapsed in the shower. A lumbar
puncture is done. Culture plates containing gram-negative bacteria show motile trophozoites
leaving trails. This patient most likely obtained the infection through which means?

A. Tick bite
B. Mosquito bite
C. Swimming
D. Drinking contaminated water
E. Penetration of intact skin from soil

53. Lymphocytes that mature in the thymus should have which of the following marker(s)?

A. CD3, CD4, CD8


B. CD14, CD16
C. CD19, CD20, CD21
D. CD40
E. CD56
54. A 68-year-old man visits his physician complaining of nausea and fatigue. He has not eaten
much in the past few days. Physical examination shows swelling in the ankles and feet. A
kidney function test is performed and the following measurements of inulin are made.

Plasma concentration: 176 mg/mL


Urine concentration: 96 mg/mL
Urine flow: 60 mL/min

What is this patient’s clearance rate in liters/day?

A. 47
B. 69
C. 90
D. 130
E. 327

55. A 76-year-old man is brought to the physician because of confusion and vision changes. He
is seeing yellow halos around objects. His past medical history is significant for chronic heart
failure, but he cannot remember his prescriptions. The most likely drug this patient is taking
has what effect on the heart?

A. Increase heart rate


B. Increase contractility
C. Increase muscle relaxation
D. Decrease afterload
E. Decrease oxygen demand

56. A 35-year-old woman visits her physician due to vision changes. She has been running red
traffic lights recently. ​Immunofluorescent imaging shows that she cannot recognize red
numbers with a green background. ​Her past medical history is significant for tuberculosis.
The drug most likely causing this patient’s symptoms prevents growth of ​Mycobacterium
tuberculosis​ through which mechanism?

A. Inhibits mycolic acid synthesis


B. Inhibits nucleic acid synthesis
C. Inhibits arabinogalactan synthesis
D. Inhibits protein synthesis
E. Accumulation of pyrazinoic acid

57. An Asian woman and African man comes to the physician for preconception counseling.
They both have α-thalassemia trait. The woman has ​cis​ deletions and the man has ​trans
deletions. They are especially concerned about having a miscarriage. What are the chances
the couples next pregnancy will result in spontaneous abortion with hydrops fetalis?
A. 0%
B. 12.5%
C. 25%
D. 33.3%
E. 50%

58. A three-day-old male newborn is brought to the pediatrician because of lethargy and
convulsions that started a day ago. The pregnancy was uncomplicated. A head CT scan
reveals cerebral edema. Serum lab values are shown.

Ammonia: 1,134 ​µmol/L (normal 100-200 µmol/L)


Glutamine: 930 mol/L (normal 420-730 micro mol/L)
Urea: 2.1 mg/dL (normal 5-18 mg/dL)
Orotic acid: Increased

What dietary changes should be recommended for this patient?

A. High fat soluble vitamin diet


B. High water soluble vitamin diet
C. Low fat diet
D. Low carbohydrate diet
E. Low protein diet

59. A 61-year-old chess grandmaster is dragged to the physician by his wife because of recent
mood changes. He has a habit of drinking heavily in celebration when winning a tournament.
However, he has been losing a lot of chess matches lately because he confuses his king
with his queen. He also tends to be more disruptive than usual when losing. Lactulose is
prescribed and his games and mood improve. What is the most likely diagnosis?

A. Alzheimer’s disease
B. Schizoaffective disorder
C. Normal pressure hydrocephalus
D. Noncommunicating hydrocephalus
E. Hepatic encephalopathy

60. A 26-year-old female immigrant from Africa comes to the physician because of abdominal
pain. Her temperature is 38.7 ​°C​ (101.7 ​°F), with a pulse of 82/min and a blood pressure of
111/79 mmHg. She had taken chloroquine before arriving in the United States a week ago.
Physical examination shows a palpable mass in the left upper abdominal quadrant. A
peripheral blood smear is shown.
Mefloquine and primaquine are given and her symptoms improve. This patient’s symptoms
are most likely caused by which of the following organisms?

A. ​Plasmodium vivax
B. ​Plasmodium ovale
C. ​Plasmodium malariae
D. ​Plasmodium falciparum
E. ​Plasmodium knowlesi

61. A 21-year-old woman comes to the physician complaining of an inability to move her arms
and legs. She was rushed to the hospital by emergency medical services 20 minutes ago
after cliff diving. She is alert, oriented, and does not require a ventilator. This patient’s spinal
cord is most likely severed at which of the following vertebral levels?

A. C1
B. C3
C. C5
D. C7
E. T1

62. An investigator is experimenting with the ability of drug X to increase heart rate. She
anesthetizes 100 mice that have denervated hearts. The drug is given intravenously at a
continuously increasing dose. Two dose-response curves are plotted.
What is the therapeutic index (TI) of drug X?

A. ½
B. 2
C. 6
D. 7
E. 100

63. A 21-year-old man visits his opthamologist because of a sudden loss of vision in his left eye.
He was playing golf with his father two days ago when he noticed his left vision was blurry.
The next morning, vision in his left eye was completely lost. He says that both his mother
and grandmother lost their vision one eye after the other. Fundoscopic examination of the
left eye reveals enlarged peripapillary vessels. There is no pupillary response in the left eye.
Direct pupillary constriction in the right eye is slow. A family pedigree of the patient (IV-4) is
shown below.

What term best explains why I-2 and II-3 are not affected?

A. Pleiotropy
B. Allelic heterogeneity
C. Locus heterogeneity
D. X inactivation (lyonization)
E. Heteroplasmy

64. A 19-year-old man visits his physician with a deep 5 cm (2 in.) laceration on the
mid-posterior region of his right arm. He was involved in a gang fight 30 minutes ago. The
wound bleeds profusely. Physical examination reveals he cannot extend his right arm at the
elbow or extend his right hand at the wrist. An X-ray of the humerus shows no fractures. If
left untreated, what is the likely fate of this patient’s limb in the pursuing months?

A. Infarction due to vessel coagulation


B. Infarction due to fat embolism
C. Permanent paralysis due to nerve death
D. Permanent paralysis due to wallerian degeneration
E. Full recovery with scar formation

65. A seven-year-old girl is brought to the physician by her father because of abnormal
behavior. Yesterday morning she was complaining of a headache. By the evening she
complained of hearing voices and refused to eat or drink. She has not slept in 48 hours.
Physical examination shows two sets of scars on the right forearm that resemble bite marks.
The most likely causative agent entered this patient’s cerebrum through which of the
following transport molecules?

A. Dynein
B. Kinesin
C. Albumin
D. Transferrin
E. Hemoglobin

66. A 13-year-old girl in her first trimester of pregnancy is brought to the gynecologist by her
father for an abortion. The father claims that an abortion is the responsible decision because
her boyfriend has a family history of Huntington's disease. He says that it is unfair to bring
an infant into this world at risk of a fatal disease. The girl clearly expresses her wish to keep
the pregnancy. What should the physician do?

A. Perform the abortion because the father is the legal gaurdian.


B. Perform the abortion because the risk of Huntington’s disease is real.
C. Perform the abortion because society has an interest in preventing teen pregnancy.
D. Do not perform the abortion because it is the girl’s choice.
E. Do not perform the abortion because the fetus is a human being.

67. A 55-year-old-man is brought to the hospital by ambulance unresponsive. His chest was
crushed against a wall by a delivery truck backing up. He is subsequently put on a ventilator
for breathing assistance. The patient’s neighbor comes to the physician with a written letter
designating her as his proxy. It is signed by the patient. She says that she does not think the
patient would want to be on a ventilator under any circumstances. The patient’s family
requests that all measures be taken to save his life, but doing so requires the ventilator.
What should the physician do?

A. Remove the ventilator because the patient has appointed his neighbor to decide for him.
B. Remove the ventilator because it is the patient’s wish.
C. Keep the ventilator because it is the only way to fulfill the family’s request.
D. Keep the ventilator because it is the physician’s duty to save lives.
E. Call the ethics committee.

68. In the year 2018, a total of 25,747 American medical students took the USMLE step 1 exam.
The average score is 231 with a standard deviation of 20. Assuming a normal bell curve,
approximately how many American medical students scored above 291?

A. 1
B. 9
C. 38
D. 196
E. 643

69. A 15-year-old girl comes to the physician because of fatigue. She was airlifted to the
hospital 30 minutes ago after being found unresponsive by a search and rescue team in the
Rocky Mountains. The girl had been missing for 14 days after being separated from her
parents on a hiking trip. Her blood pressure is 103/69 mmHg, pulse 90/min, respirations
28/min. Physical examination reveals a disheveled, underweight teenager with bad breath.
Serum laboratory values are shown.

Sodium: 150 mEq/L


Potassium: 4 mEq/L
Chloride: 111 mEq/L
Bicarbonate: 19 mEq/L
PaCO₂: 36 mmHg
pH: 7.31

What is this patient’s acid base balance?

A. Respiratory acidosis compensated


B. Respiratory acidosis uncompensated
C. Respiratory alkalosis compensated
D. Respiratory alkalosis uncompensated
E. Metabolic acidosis compensated
F. Metabolic acidosis uncompensated
G. Metabolic alkalosis compensated
H. Metabolic alkalosis uncompensated
I. Mixed acid-base disturbance

70. A 29-year-old woman visits her physician because of fever, night sweats, productive cough,
fatigue, muscle pain, and joint pain. Her symptoms started 6 days ago after she came home
from a business trip in the American southwest. Her temperature is 38 °C (100.4 °F), pulse
95/min, and blood pressure 120/70 mmHg. Physical examination shows a lesion on the neck
indicated by the photo.
Lab results show her CD4 count to be 400 cells/µL. A drug extracted from ​Streptomyces
nodosus​ is given intravenously. What is the mechanism of action of this drug?

A. Interact with ergosterol to form membrane pores


B. Prevents ergosterol synthesis by inhibiting lanosterol 14 α-demethylase
C. Prevents formation of squalene epoxide by inhibiting squalene epoxidase
D. Form deposits in keratin and interfere with microtubule structure
E. Inhibits synthesis of β-1,2 glucan

71. A 42-year-old man comes to the physician after the Centers for Disease Control and
Prevention (CDC) sent him a warning letter that his ex-boyfriend tested positive for human
immunodeficiency virus (HIV). An enzyme linked immunosorbent assay (ELISA) of the
patient’s serum showed an antibody titer of 1/64. Further testing with polymerase chain
reaction (PCR) confirms the diagnosis of HIV infection. Enfuvirtide and maraviroc are
started. Maraviroc prevents the binding of HIV protein gp120 to CCR5 on a set of CD4+
cells. What is the identity of these cells?

A. Double positive T cells (CD4+/CD8+)


B. Helper T cells
C. Regulatory T cells (Tregs)
D. Macrophages
E. Dendritic cells

72. What is a chromosome made up of?

A. Acids and fats


B. Acids and proteins
C. Steroids and proteins
D. Proteins and fats
E. Acids, steroids, and complex sugars
73. A 23-year-old woman visits her physician because of complications with chewing. She was
involved in a catfight at a local bar two days ago. During the altercation, her adversary
grabbed her face and dug with acrylic nails into the sides of her cheeks. Physical
examination shows four deep dent marks on either side of the face. Two marks are visible
just below the greater wing of the sphenoid bone on each side, and two marks are visible
anterior to the temporomandibular joint on each side. Through further inquiry, she claims to
yawn with clenched teeth. Which pair of muscles are most likely damaged in this patient?

A. Masseter
B. Temporalis
C. Medial pterygoid
D. Lateral pterygoid
E. Digastric (anterior belly)

74. An 11-month-old boy is brought to the physician by his mother after he fell on outstretched
hands while standing. Physical examination shows multiple bruises on the arms and legs.
His sclera has a blue hue. An X-ray of the left arm is shown.

​Courtesy of ​Wikimedia Commons and Shakata GaNai

Additional X-rays reveal bowing of both femurs, with a greenstick fracture of the left femur.
What should the physician do?

A. Tell the mother to make sure her child has a healthy lifestyle and avoid aggressive sports.
B. Ask the mother if there has been any child abuse.
C. Admit the child to separate him from any more potential harm.
D. Inform the mother that she is being reported to child protective services.
E. Call the police.

75. A scientist is studying the effects of colchicine on leukemia. He draws 10 mL of blood from a
leukemic patient and adds colchicine to the sample. After overnight incubation, the white
blood cells of the sample are fixed with methanol and glacial acetic acid. Under microscopic
examination, numerous cells are observed in metaphase. There are no cells in anaphase.
Colchicine inhibits mitosis through what mechanism?
A. Cross links DNA strands
B. Intercalates between base pairs
C. Inhibits spindle fiber formation
D. Inhibits dihydrofolate (DHF) reductase
E. Inhibits thymidylate synthase

76. A 48-year-old man takes his wife to the genetic counselor because he suspects his wife has
been cheating on him. Both of them have osteogenesis imperfecta (OI), yet none of their five
children have symptoms. Their family pedigree is shown.

A paternity test “could not rule out the possibility” that the man is the biological father. What
is the most likely explanation as to why the children are unaffected?

A. Incomplete penetrance
B. Locus heterogeneity
C. Allelic heterogeneity
D. X-inactivation (lyonization)
E. Pleiotropy

77. A 69-year-old man comes to the ophthalmologist because of a sudden loss in color vision in
his left eye 30 minutes ago. He had stopped taking his medications, which include warfarin
and pravastatin, because of financial difficulties. His visual acuity is 20/20 in both eyes with
contacts. Pupils are equal, round, and reactive to light. What anatomical structure is most
likely damaged in this patient?

A. Cornea
B. Fovea
C. Optic nerve
D. Occipital lobe
E. Temporal lobe

78. An 18-month-old girl is brought to the pediatrician by her father because of abnormal
speech. She can repeat phrases and speak in short sentences, but the words are barely
discernible. Hypernasal voice resonance is determined to be the cause. A picture of her
nose and mouth is shown.

​Courtesy of Dr. Youssef Tahiri

A failure of fusion of which two embryonic structures most likely resulted in the child’s
physical abnormality shown?

A. The medial nasal prominence with the lateral nasal prominence


B. The medial nasal prominence with the maxillary prominence
C. The lateral nasal prominence with the maxillary prominence
D. The maxillary prominence with the mandibular prominence
E. The frontonasal prominence with the mandibular prominence

79. A 27-year-old primigravida woman gives birth to a newborn girl. The infant is cyanotic upon
delivery. An endocardiograph shows Tetralogy of Fallot. What is the first developmental
abnormality that leads to Tetralogy of Fallot in this infant?

A. Right ventricular hypertrophy


B. Patent ductus arteriosus
C. Pulmonary stenosis
D. Ventricular septal defect (VSD)
E. Overriding aorta

80. ApoB100 is a protein made by the liver encoded by the ​AOPB​ gene. The exact same gene
in the intestines makes the ApoB48 protein. In the liver, mRNA residue 2153 contains CAA,
whereas in the intestines, mRNA residue 2153 contains UAA. ApoB48 contains the first 48%
of ApoB100. What mRNA modification accounts for the difference in protein length?

A. Alternative splicing
B. O-linked glycosylation
C. Cytosine deamination
D. Phosphorylation of mannose
E. mRNA cleavage by argonaute

81. A 50-year-old homeless man is brought to the physician unresponsive after being found
lying on the street with an empty bottle of vodka. He is declared deceased on arrival. An
autopsy is performed. His liver is shown below.
​Courtesy of ​Clara Rodríguez Fernández

This patient’s liver has which of the following abnormal conditions?

A. Hepatitis
B. Cirrhosis
C. Fibrosis
D. Ischemic nutmeg
E. Hemorrhage

82. A 30-year-old primigravida woman at 32 weeks gestation visits her OB/GYN for routine
prenatal care. She has no complaints and appears healthy. A measurement from the pubic
bone to the fundus of the uterus shows 32 cm. What is/are the primary sites of
hematopoiesis in the fetus?

A. Yolk sac
B. Liver and spleen
C. Liver and bone marrow
D. Liver and lymph nodes
E. Lymph nodes

83. An 81-year-old woman with chronic heart failure is prescribed sacubitril. This drug works by
inhibiting neprilysin, an enzyme that degrades both atrial natriuretic peptide (ANP) and
angiotensin II. Knowing this information, what drug should also be given to this patient?

A. Furosemide
B. Verapamil
C. Hydralazine
D. Valsartan
E. Lidocaine

84. ​A 12-year-old boy is brought to the physician by his mother because of learning difficulties at
school. Physical examination shows no abnormalities. A blood sample is drawn and sent for
cytogenetic analysis. A week later, the results show that 40% of his white blood cells are
positive for trisomy 21. When did nondisjunction most likely occur in this patient?

A. Meiosis I
B. Meiosis II
C. Mitosis in early embryogenesis
D. Mitosis in the bone marrow after birth

85. A 27-year-old man comes to the physician because of numbness and tingling on the medial
aspect of his left arm and forearm that has persisted for the past 40 days. He is a right
handed professional baseball pitcher. A radial pulse in the left wrist is barely palpable.
Physical examination shows weakened thumb adduction and weakened extension of
interphalangeal digits 4 and 5 on the left. Atrophy of the hypothenar eminence is also
observed. What is the most likely cause of the patient’s symptoms?

A. Medial wrist laceration


B. Fracture of the hook of hamate
C. Constriction of Guyon’s canal
D. Trauma to the medial epicondyle
E. Hypertrophy of the scalene muscles.

86. A new screening test is developed in Connecticut for lyme disease where its prevalence is
high. Out of a sample of 100 individuals screened for the disease, 20 tested positive. Those
same 100 individuals then underwent a diagnostic test for ​Borrelia​ antibodies using ELISA.
ELISA confirms 25 individuals have lyme disease which includes 15 individuals who
screened positive. What is the sensitivity of the new screening test?

A. 60%
B. 75%
C. 85%
D. 87.5%
E. 93.3%

87. A group of twenty football players planned a barbecue in a private backyard. It began to rain
so the barbecue was held in the basement instead. Fifteen players were present in the
basement and five players remained on the patio drinking beer. Later that evening, seven
players were brought to the hospital with nausea and vomiting and two unfortunately died.
What is the attack ratio?

A. 2/20
B. 2/15
C. 2/7
D. 7/20
E. 7/15
88. A seven-year-old boy is brought to the physician by his father because of frequent
screaming in the middle of the night. He wakes up his parents most nights and sometimes
has multiple episodes per night. When asked, the boy has no memory of what awakes him.
He only remembers being afraid. Diazepam is given and his symptoms subside. What is the
most likely diagnosis?

A. Insomnia
B. Somnambulism
C. Narcolepsy
D. Night terror
E. Nightmare

89. A 15-year-old girl comes to the physician with her mother for a physical as part of a
requirement for soccer tryouts at her local high school. She is cheerful and has no
complaints. During the physical the physician notices a long linear scar on the back of the
girl. The girl is surprised at the finding and says she thinks it might be a birthmark. Later the
mother says the scar was a result of a beating that happened years ago. What defense
mechanism is this girl most likely exhibiting?

A. Denial
B. Dissociation
C. Blocking
D. Repression
E. Suppression

90. Two researchers are conducting an experiment to investigate whether fast food
consumption is related to increased blood pressure. They are able to round up 1,100 male
participants 18 years of age with similar blood pressure and BMI. Out of these participants,
100 report that they enjoy eating fast food regularly and are separated out from the
remaining 1,000. Ten years later it is found that 60 of the 100 participants in this group
developed high blood pressure compared to 200 of the remaining 1,000 participants from
the other group. Based on this study, how many times more likely is it for a fast food eater to
develop high blood pressure compared to a non-fast food eater?

A. 1.5x
B. 2x
C. 2.7x
D. 3x
E. 5x

91. A 25-year-old medical student from China is on surgical rotation. He is known by his peers
to be prejudiced toward Japanese nationals. His mind is often occupied by historical
conflicts between China and Japan. He once told a peer “I just want to tear those Japanese
bodies to shreds.” During his surgical rotation he is particularly attentive and eager to learn.
The attending physician compliments him on his enthusiasm in which the student replies, “I
always wanted to be a trauma surgeon.” Which defense mechanism is this student
demonstrating?

A. Displacement
B. Acting out
C. Sublimation
D. Isolation of affect
E. Intellectualization
F. Splitting

92. A 42-year-old woman gravida 3, para 2, comes to the gynecologist. She is 10 weeks
pregnant and concerned that her advanced maternal age puts her unborn child at increased
risk of chromosomal abnormalities. A transcervical chorionic villus sample (CVS) is taken
and sent for cytogenetic analysis. The results of the analysis is shown.

What is the most likely fate of the embryo?

A. Spontaneous abortion (miscarriage)


B. Severe intellectual disability
C. Early Alzheimer’s disease
D. Death by age 1
E. Normal male development

93. A 55-year-old man visits his urologist for prostate screening. He is late for his visit because
he was stopped and ticketed for speeding on the way to the appointment. During the exam,
he strikes up a casual conversation with the physician in which he mentions he wants to kill
the ticketing officer for making him late. What is the appropriate course of action?

A. Try to detain the patient, call the police department, and contact the ticketing officer.
B. Only call the police department because it is their professional responsibility to handle
this situation, not the physician’s.
C. It is only necessary to contact the ticketing officer because it is a personal matter
between him and the patient.
D. Do not mention the conversation to anyone without the patient's consent because it
would be a violation of patient privacy.
E. It is small talk and the patient’s statements should not be taken seriously.

94. An 11-year-old boy is brought to the pediatrician by his parents for an annual well-child
examination. During the exam, the boy tells his pediatrician that the persistent accumulation
of smegma between his foreskin and glans penis feels unsanitary and causes him
psychological discomfort. He requests a circumcision. The father refuses to grant permission
because he considers the surgery unnatural. The mother says she is okay with the boy
having the surgery. What should the physician do?

A. Give the surgery because the boy is in discomfort.


B. Give the surgery because the mother grants permission.
C. Do not give the surgery because it is not a life threatening emergency.
D. Do not give the surgery because the father refuses permission.
E. Call in the ethics committee to resolve the disagreement between the parents.

95. An 84-year-old retired Oregon state judge is in hospice care with terminal esophageal
cancer. She is intubated because an esophageal tumor is compressing her trachea. She
asks her geriatrician to remove her tube and prescribe her 10 grams of liquid pentobarbital
to end her life because she is suffering. She reminds the geriatrician that under Oregon’s
Death with Dignity Act of 2002, her request is fully legal. The patient appears to have full
capacity. What should the geriatrician respond?

A. “I will grant your request fully.”


B. “I can remove your tube, but I cannot ethically participate in assisted suicide.”
C. “I can prescribe you the medication but I must keep you intubated.”
D. “I understand your pain, would you like morphine instead?”
E. “I will call the ethics committee and have them decide.”

96. A 66-year-old man comes to the physician with hemoptysis, dyspnea, and chest pain. His
symptoms began 20 minutes ago while seated in the passenger seat of a car. His pulse is
90/min, blood pressure 150/100 mm Hg, breaths 35/min short and shallow. A
ventilation/perfusion lung scan shows a result of 1.8 and an ELISA test reveals elevated
D-dimers. What is the most likely diagnosis?

A. Coronavirus (COVID-19)
B. Myocardial infarction (MI)
C. Pulmonary embolism (PE)
D. Paradoxical embolism with Cheyne-Stokes breathing
E. Transient ischemic attack (TIA)

97. A young African American couple visits their genetic counselor. They are planning to take
their newborn son to Africa to visit relatives. However, the mother is only comfortable with
the trip if her son has natural resistance to malaria like herself. The mother is confirmed to
have sickle cell trait, but the carrier status of the father is unknown. If the prevalence of
sickle cell disease in African American men is one in 576, what is the percent chance that
the couple’s son has resistance to malaria?

A. 1%
B. 4%
C. 9%
D. 25%
E. 50%

98. A 54-year-old man is brought to the physician after being found lying on the street
unresponsive. His breath smells of alcohol. Intravenous dextrose is administered. A few
hours later the patient expires due to complications of vomiting and respiratory distress. A
root cause analysis reveals that thiamine (B​1​) was not given prior to dextrose despite the
physician’s knowledge of the patient's condition and appropriate treatment procedure. The
physician has made an error of which category?

A. Slip
B. Memory lapse
C. Mistake
D. Violation
E. Near-miss

99. A 17-year-old local high school track sprinter is brought to the physician after she collapsed
in a midway sprint during practice. She is declared dead on arrival. Her biological father died
of sudden death a few years ago. An autopsy of the girl reveals a heart with an enlarged left
ventricle (LV). What is the likely pathophysiologic sequence leading to the death of this
patient?

A. Dilation of the LV→ ↓ contractility → ↓ cardiac output


B. Thickening of the LV wall → ↓ compliance → ↓ filling → ↓ cardiac output
C. Thickening of the LV septum → cardiac outflow tract collapse → ↓ cardiac output
D. Fibrosis of the LV wall → scaring → ↓ contractility → ↓ cardiac output
E. Prinzmetal angina of the left anterior descending artery (LAD) → LV wall infarction →
rupture of LV → pericardial effusion → ↑ cardiac burden → ↓ cardiac output

100. A 59-year-old man comes to the physician because of a three month history of coughs with
copious amounts of malodorous sputum production. He has smoked two packs of cigarettes
a day for the past 35 years. Physical examination shows a large ribcage with a round torso.
A chest x-ray reveals a flattened diaphragm with evidence of damage to proximal acini. A
sputum culture grows golden yellow colonies on blood agar. What are the most likely results
on pulmonary function testing for this patient?

Total lung Forced Forced vital FEV​1​/FVC Peak flow Functional Residual
capacity expiratory capacity residual volume
volume per (FVC) capacity
second
(FEV​1​)

A. ↓ ↓ ↓ ↔ ↓ ↓ ↓

B. ↑ ↓ ↔ ↓ ↓ ↑ ↑

C. ↑ ↓ ↑ ↑ ↑ ↔ ↔

D. ↓ ↑ ↓ ↔ ↑ ↓ ↔

E. ↔ ↔ ↓ ↔ ↓ ↓ ↓

101. A virologist is studying two different viruses. Virus A and virus B look very different under an
electron microscope. Alcohol is applied to both viruses. Virus A looks the same as before.
Virus B now looks like virus A. What is a key difference between virus A and virus B after
applying alcohol?

A. Virus B is osmotically unstable


B. Virus B is structurally unstable
C. Virus B is more easily recognized by immune cells
D. Virus B cannot enter host cells
E. There is no difference

102. ​A 63-year-old man comes to the physician because of fatigue and a persistent dry cough
that has lasted for 6 weeks. He is often short of breath after climbing the stairs to his
apartment. His blood pressure is 142/93 mm Hg, pulse 65/min, and respirations 23/min.
Physical examination reveals clubbing of the fingers. A pulmonary function test shows an
increased forced expiratory volume to forced vital capacity ratio (​FEV​1​/FVC). A needle
biopsy of the lung is shown.
​Courtesy of Agnès Gardet, Timothy S. Zheng, Joanne L. Viney

What is the most likely diagnosis?

A. Tuberculosis
B. Chronic bronchitis
C. Emphysema
D. Sarcoidosis
E. Idiopathic pulmonary fibrosis

103. A 23-year-old man comes to the physician because of a two-week history of cramping
abdominal pain followed by bloody diarrhea. He has been feeling fatigued with a six pound
unintentional weight loss in the past month. Physical examination shows multiple
hyperpigmented flat pinpoint lesions on the upper and lower lips. He claims that his mother
and five sisters all have similar lesions. Family history is only significant for breast cancer in
his mother. A barium swallow test followed by a barium enema radiograph reveals dozens
of polyps from the esophagus to the sigmoid colon. A biopsy of a large mass at the splenic
flexure confirms colon cancer. This patient most likely has what inherited condition?

A. Li-Fraumeni syndrome
B. Hereditary hemorrhagic telangiectasia
C. Peutz-Jehgers syndrome
D. Familial adenomatous polyposis
E. Lynch syndrome (HNPCC)

104. A seven-year-old girl is brought to the pediatrician because of a fever and decreased
energy level. Her father says that she has not been acting like herself lately. Her
temperature is 38.7 °C​ ​(101.7 °F), pulse is 65/min, and blood pressure is 130/85 mm Hg.
Past medical history reveals a case of strep throat three weeks ago that was treated with
antibiotics. Physical examination shows periorbital edema. When asked, the girl says she
doesn't feel like urinating as often as she used to. A serum ELISA test demonstrates an
anti-streptolysin O antibody titer of 1:256. Which organ is most likely damaged in this
patient?
A. Brain
B. Heart
C. Kidney
D. Adrenal medulla
E. Adrenal cortex

105. A 58-year-old man comes to the physician because of a 24 hour history of pain in the
abdomen and back. He has had a decrease in appetite accompanied by an 8.6 kg (19 lbs)
unintentional weight loss in the past month. Past medical history and family history are
insignificant. Physical examination shows jaundice with scleral icterus. Palpation of the right
upper quadrant of the abdomen reveals a nontender mass. A serum laboratory test reveals
an increased direct (conjugated) bilirubin level. The patient is subsequently admitted to the
hospital, but unexpectedly expires the following morning. A cross section of the liver at
autopsy is shown.

What is the most likely diagnosis?

A. Hepatic cirrhosis
B. Hepatic adenoma
C. Hepatoblastoma
D. Hepatocellular carcinoma
E. Stage IV (metastatic) pancreatic cancer

106. A researcher in a laboratory is given a critical mathematical formula required for an


experiment. He cannot read the formula because of its poor handwriting. However, the
units can be read. The units are m/s, mm​2​, and mm​3​/s. He puts a letter in front of each unit
as follows: X mm/s, Y ​mm​2​, Z ​mm​3​/s. Based on this information, what must the formula be?

A. X=YZ
B. X=Y/Z
C. X=Z/Y

107. A 33-year-old man is brought to the physician unresponsive. Physical examination shows a
large area of discoloration at the angle of the jaw. Spontaneous breathing is observed, but
painful body stimulus fails to elicit a response. Voluntary eye movements are absent, yet
applying cold water into the external ear canal causes the eyes to shift to the opposite side.
Laboratory tests are pending. What is the most likely diagnosis?

A. Locked-in syndrome
B. Coma
C. Persistent vegetative state
D. Brain death

108. A 65-year-old woman with refractory multiple myeloma comes to the physician for a
scheduled bone marrow transplantation. The patient’s healthy brother and sister are equally
compatible donors and both agree to donate. Why is this patient’s brother the preferred
donor?

A. Male donor marrow are less likely to be rejected by the host


B. Male donor marrow is less likely to cause graft-versus-host disease (GvHD)
C. Male donor marrow is statistically less likely to have multiple myeloma recur.
D. Donor and recipient marrow are more easily distinguished for monitoring purposes

109. A 67-year-old recent black immigrant woman comes to the physician because of back pain
that has been increasing in frequency and severity for the past few weeks. She was raised
in Kenya but has lived in Sweden for most of her adult life. Her past history includes pain in
her wrists and hip. Physical examination shows no abnormalities. An X-ray reveals a
compression fracture at the L3 vertebrae. Additional X-rays reveal transverse lucencies
close to the surgical necks of the humeri and femurs. Laboratory results show the following:

Hemoglobin 12.5 g/dL


Hematocrit 37%
Leukocyte count 4,800/mm​3
Serum
Calcium (Total) 8.0 mg/dL
Phosphorus 2.7 mg/dL
Alkaline phosphatase 73 U/L

What is the likely diagnosis?

A. Osteoporosis
B. Osteomalacia
C. Osteoarthritis
D. Rheumatoid arthritis
E. Reactive arthritis

110. A new computerized screening test for COVID-19 is being evaluated for its sensitivity. Five
participants confirmed to be infected and 95 participants confirmed to not be infected are
run through the computer program. All 100 participants received a positive result. A glitch in
the computer only allowed a positive result to be given. What is the sensitivity of the
computerized screening test with the glitch present?

A. 0%
B. 5%
C. 95%
D. 100%
E. It cannot be determined

111. An archaeologist picked up a peculiar object at an archaeological site. Laboratory


examination reveals the object is an ossified part of a human body derived from neural
crest origin. What part of the body did the object most likely come from?

A. Head
B. Torso
C. Hip
D. Arm
E. Leg

112. A pair of dizygotic twins are born with craniofacial abnormalities and heart defects. Blood
samples of the twins are sent for cytogenetic analysis to confirm the diagnosis. Three days
later, the results come back. One report shows 47,XX,+21 and the other report shows
46,XY,rob(14;21)(q10;q10),+21. What conclusion can be made about the twin patients?

A. The boy has sporadic disease. The girl has inherited disease.
B. The boy has inherited disease. The girl has sporadic disease.
C. Both have sporadic disease.
D. Both have inherited disease.
E. They are monochorionic-diamniotic twins.

113. A 13-year-old boy presents to the emergency department with testicular pain 30 minutes
after getting kicked in the groin during a fight at school. The right scrotum appears swollen.
The physician gently strokes the superior medial right thigh of the patient. The right testicle
elevates. Which nerve root is involved in this response?

A. L1
B. L5
C. S1
D. S2
E. S4

114. A couple comes to the clinic for genetic counseling. They have recently learned that they
are blood relatives. The counselor informs the couple that consanguinity is associated with
a vast array of diseases. Out of the list of options, which disease is not associated with
consanguinity?

A. Hemochromatosis E. Cystic fibrosis


B. Wilson disease F. Friedreich ataxia
C. Phenylketonuria G. Sickle cell anemia
D. Alkaptonuria H. Achondroplasia

115. A 31-year-old primigravida woman comes to the clinic at 28 weeks gestation for a routine
prenatal exam. Previous lab results showed she is Rh​-​ with a Rh​+​ fetus. The attending
physician orders an administration of RhoGAM. The resident delivers 50 μg of the drug via
the intramuscular route on the upper inner quadrant of the right buttocks. One hour later the
patient returns to the clinic complaining of shooting pain in her back that radiates down her
right posterior thigh and leg. Which nerve is most likely damaged in this patient?

A. Superior gluteal nerve


B. Inferior gluteal nerve
C. Sciatic nerve
D. Obturator nerve
E. Femoral nerve

116. A 42-year-old man wakes up from a 10 month coma after a severe case of herpes simplex
encephalitis. When the physician asks for his name and place he responds appropriately.
When he is told the date, he asks if he was in a coma. The patient's speech appears to be
fluent. The physician leaves the room to retrieve a brain scan of the patient. When she
returns, he does not recognize her and does not have any recollection of their previous
interaction. Which area of the brain is most likely damaged in this patient?

A. Broca’s area
B. Wernicke’s area
C. Superior longitudinal fasciculus
D. Inferior longitudinal fasciculus
E. Hippocampus
F. Amygdala
G. Cerebral cortex

117. A healthy baby boy was born an hour ago. His ABO blood type is unknown. If the allele
frequency of A is 32%, B is 23%, and O is 45% for the community, what is the probability
he has type B blood?

A. 11%
B. 21%
C. 26%
D. 34%
E. 41%

118. A 12-year-old girl comes to the physician because of a week of dysphagia and dyspnea.
She has recently started jogging with her friends but is easily out of breath early in the run.
Physical examination shows no abnormalities except for a moderate mass at the base of
the tongue. Biopsy and histology reveals thyroid tissue. If this mass is the full thyroid gland,
how does it differ from a normally placed thyroid?

A. It is completely nonfunctional
B. It is more sensitive to thyroid stimulating hormone
C. It produces a relatively higher T​3​/T​4​ ratio
D. It lacks parafollicular C cells
E. It is at greater risk for hyperactivity

119. A 19-year-old man with Down syndrome and acute lymphocytic leukemia is brought to the
physician by his mother for a scheduled round of chemotherapy. This will be the fourth time
the patient is given injection drugs. Prior to the start of treatment, the patient tells the
physician that he no longer wishes to continue chemotherapy because it is physically
painful and makes him feel sick. The physician carefully explains to the patient that the
chance of remission is high with continued chemotherapy and death is almost certain
without it. The patient clearly acknowledges that he understands the risks and benefits of
both continuing and discontinuing treatment, and that there are no adequate alternatives,
yet he still requests discontinue. The patient’s mother insists that therapy be continued
because he has Down syndrome. What is the best course of action?

A. Discontinue chemotherapy because the patient understands the risk and benefits
B. Discontinue chemotherapy because it is causing the patient physical pain and discomfort
C. Continue chemotherapy because the patient has Down syndrome
D. Continue chemotherapy because the patient’s mother insists
E. Call the ethics committee

120. Twin infant girls are born at 38 weeks gestation in uncomplicated vaginal deliveries.
Ultrasound had shown one chorion and two amnions prior to birth. The first newborn
appears normal. The second newborn displays a webbed neck, low set ears, widely spaced
nipples, and a narrow palate. Blood karyotyping reveals normality in the first twin, but the
second twin is diagnosed with mosaic Turner syndrome. The relationship between the twin
patients can best be described as:

A. Monozygotic (Identical)
B. Dizygotic (Fraternal)
C. Conjoined
D. Parasitic
121. A new screening test is developed to identify individuals with coronavirus disease of 2019
(COVID-19). This test uses a thermometer to detect temperatures ≥100℉ (37.8℃),
indicating a positive result. Patients who test positive then undergo polymerase chain
reaction (PCR) testing to confirm the result. A report of 1,000 patients who underwent this
screening test revealed a specificity of 67%. What does this percentage represent?

A. The probability that a patient with the disease tests positive.


B. The probability that a patient who tests positive has the disease.
C. The probability that a patient without the disease tests negative.
D. The probability that a patient who tests negative does not have the disease.
E. The probability that a patient who is tested is correctly identified as having the disease or
not having the disease.

122. A 69-year-old woman comes to the physician due to a two day history of unilateral hearing
loss and pain in her left ear. She had an upper respiratory tract infection six weeks ago that
resolved with the use of amoxicillin and clavulanate. Past family history is unremarkable.
The patient eats a balanced diet and swims at her local community pool three times a
week. She has smoked a pack of cigarettes daily for the past 35 years but does not use
alcohol, or illicit drugs. Her temperature is 37.83℃ (100.1℉), pulse is 78/min, and
respirations are 18/min. Physical appearance is normal except for a purulent discharge
coming from her left ear canal. Otoscopic examination using a rubbery bulb attached to the
otoscope reveals an immobile tympanic membrane. Culture of the exudate grows
gram-negative diplococcus on chocolate agar. How did the infecting pathogen most likely
enter this patient’s ear?

A. Through the external auditory meatus


B. Through the Eustachian tube
C. From rupture of the round window membrane
D. From fracture of the floor of the middle cranial fossa

123. The X chromosome is unique because it is the only chromosome that can inactivate. This
special ability is due to the X-inactive specific transcript (XIST) gene located on Xq13. On
the other hand, Alport syndrome is a X-linked dominant disease that causes vision, hearing,
and renal impairments. If a female child is conceived from the union of a XIST mutated X
chromosome and a X-linked dominant Alport syndrome X chromosome, what would be the
likely outcome?

A. A girl with Alport syndrome


B. A girl without Alport syndrome
C. A girl with a milder form of Alport syndrome
D. It is hard to predict due to random X-inactivation

124. A 67-year-old man comes to the physician due to progressive hearing loss for the past two
years. He is a retired lumberjack, but still does woodwork around his house with heavy
equipment. His past medical history and family history are unremarkable. He does not use
tobacco, alcohol, or illicit drugs. A vibrating tuning fork is placed on the patient’s right
mastoid process until he can no longer hear the sound. Then the tuning fork is immediately
placed in front of the ipsilateral ear in which the sound can be heard for an additional two
seconds. When the tuning fork is placed on the vertex of the skull, no localization is
detected. What is the most likely cause of this patient’s hearing loss?

A. Impacted cerumen
B. Desquamated keratin debris within the middle ear cavity
C. Destruction of hair cells at the cochlear base
D. Schwannoma at the pontomedullary junction
E. Hemorrhagic infarction involving the primary auditory cortex
F. Conversion disorder
G. Factitious disorder imposed on self

125. An investigator is trying to determine the relative risk (RR) of developing type II diabetes
mellitus in association with weight. He gathers a cohort of nondiabetic participants matched
for ethnicity, sex, and age. They are then divided into three categories: normal (18.5-25
kg/m​2​), overweight (25-30 kg/m​2​), and obese (>30 kg/m​2​). The results are analyzed after a
20 year follow up. As compared to normal, the relative risk of developing diabetes in the
overweight and obese are 2.3 and 4.6 respectively. What is the relative risk of developing
diabetes in the overweight as compared to the obese?

A. 0.5
B. 1.0
C. 2.0
D. 2.3
E. 4.6

126. A 26-year-old man comes to the office for an annual physical examination. He has no
complaints. The patient is 132 cm (4’4 ft) tall and weighs 47 kg (104 Ibs), BMI 26.8 kg/m​2​.
Physical features include short limbs, a small midface with a flattened nasal bridge,
prominent forehead, and a head and torso size more consistent with an average adult
male. At the time of this patient’s birth, what piece of information was most likely present in
his family history?

A. A mother with multiple miscarriages


B. A mother with alcoholism
C. A mother with hypothyroidism
D. A father with advanced age
E. A parent with the same condition as the patient

127. The patient above has achondroplasia. If he has an offspring with a woman with the same
condition, what is the likelihood their child will also have achondroplasia?
A. 1/4
B. 1/2
C. 2/3
D. 3/4
E. 1/1

128. A 55-year-old man comes to the office because of difficulty with movement. He has been
completely wheelchair bound for the past week. Six months ago the patient visited the
physician complaining of a weakened grip that made him drop his coffee every morning. A
tentative diagnosis of multiple sclerosis was made at that time. His symptoms have now
progressed to his legs and he can no longer lift himself out of bed. He has experienced no
personality changes. The patient’s family history is unremarkable. He does not use
tobacco, alcohol, or illicit drugs. Physical examination shows weakened arm and leg
strength bilaterally. The Babiniski sign is positive. There are no vision, autonomic, or
cognitive impairments. Feelings of pain, temperature, and vibrations are intact. A drug with
what mechanism of action is likely to ease this patient’s condition?

A. Inhibits release of calcium from the sarcoplasmic reticulum of skeletal muscles


B. Inhibits vesicular monoamine transporter (VMAT) in motor neurons
C. Inhibits acetylcholinesterase (AChE) at the neuromuscular junction
D. Stimulates 𝛼​2​ receptors in the central nervous system
E. Stimulates GABA​B​ receptors in the spinal cord

129. A human sperm is on average 5 µm long and can travel up to 4 mm/min. This is equivalent
to a car traveling at 216 km/h (134 mph). The speed of the sperm is powered by organelles
located within its middle piece. What organelle is likely to be abundant in this portion of the
sperm?

A. Nuclei
B. Rough endoplasmic reticulum (RER)
C. Mitochondria
D. Lysosomes
E. Peroxisomes

130. A structure that passes through the center of the circle of Willis is damaged. What is the
most likely neurological deficit?

A. Blindness
B. Deafness
C. Ataxia
D. Hyposmia
E. Hypogeusia
131. An infant is born at 39 weeks gestation to a 41-year-old woman. The infant has two older
siblings with Down syndrome. Physical features of the infant include a short neck with
excessive skin, a furrowed tongue, brushfield spots on the irises, and sandal-toe
deformities on both feet. A blood analysis of the newborn using fluorescent in situ
hybridization (FISH) centromere probes reveal 46 chromosomes. The mother is also
revealed to have 46 chromosomes. How many chromosomes is this infant’s father most
likely to have?

A. 45 chromosomes
B. 46 chromosomes
C. 46 chromosomes with one dicentric chromosome
D. 46 chromosomes with acentric fragments
E. 47 chromosomes

132. Trisomy 21 is the most common autosomal trisomy at birth. What is the reason trisomy 21
is more common than any other autosomal trisomy (e.g. trisomy 4)?

A. Chromosome 21 is the most common to be involved in nondisjunction.


B. Chromosome 21 is small and light, therefore sperms carrying it can swim faster.
C. Other trisomies have less chance to fertilize the ovum.
D. Trisomy 21 produces the most viable embryo because chromosome 21 is small.
E. Only chromosome 21 can be involved in translocation.

133. A 22-year-old man comes to the physician due to persistent diarrhea for the past week.
Laboratory results show a normal anion gap metabolic acidosis. What condition does this
patient have based on the laboratory results?

A. Hypernatremia
B. Hyperkalemia
C. Hypercalcemia
D. Hyperchloremia
E. Hyperuricemia
F. Hypoxemia

134. A 20-year-old man is rushed to the emergency room after smashing his head on the
windshield (coup injury) of a vehicle. He was the unrestrained passenger in a head on
collision. The patient is oriented to person, place, and time. He has no serious complaints.
Physical examination indicates anosmia. A cranial CT scan reveals severe damage to the
olfactory bulbs. In six months time, what is this patient most likely to experience?

A. Coagulative necrosis of the olfactory bulbs


B. Liquefactive necrosis of the olfactory bulbs
C. Caseous necrosis of the olfactory bulbs
D. Return of smell
E. Loss of taste

135. If you look on page 90 of First Aid 2020, you will see two ketones produced, acetoacetate
and β-hydroxybutyrate. The step to convert acetoacetate to β-hydroxybutyrate seems
unnecessary as it must be converted back to acetoacetate in order to be used as fuel.
Given that NADH must be used to reduce acetoacetate to β-hydroxybutyrate, this extra
step is likely most active in which type of ketoacidosis?

A. Diabetic ketoacidosis
B. Alcoholic ketoacidosis
C. Starvation ketoacidosis

136. A 47-year-old man comes to the physician due to numbness in his right hand. The patient
is 165 cm (5’5 ft) tall and weighs 95 Kg (209 lbs), BMI 34.8 Kg/m​2​. He experiences no
weakness nor loss of sensation in either his arm or forearm. The physician cuts the
transverse flexor ligament (flexor retinaculum). Sensation is likely to return to which area of
this patient's hand?

A. Palmar aspect of the first 3½ digits


B. Lateral palm and palmar aspect of the first 3½ digits
C. Medial hand and the last 1½ digits
D. The lateral dorsum including the anatomical snuff box

137. A small community has reported an alarming increase in the number of asthma cases
within its elementary school system. A cross-sectional study is undertaken to determine the
prevalence of asthma among four different schools. A graph of the results is shown.

Which school has the highest prevalence of children diagnosed with asthma?

A. School 1
B. School 2
C. School 3
D. School 4

138. A study is called a cross-sectional study because it is analogous to which of the following
actions?

A. Monitoring the height of a growing child


B. Counting the number of customers coming into a store
C. Separating out ripe apples from rotten apples
D. Cutting a bone at its diaphysis and examining its cut surface

139. A 44-year-old man comes to the physician because of a painless enlarging mass in the
upper outer quadrant of his left breast. Scirrhous growth is seen on biopsy. The patient is
subsequently diagnosed with invasive ductal carcinoma.

Which family member of this patient (P) is most likely to develop breast cancer?

A. A
B. B
C. C
D. D
E. E

140. A young couple comes to hospital requesting heart transplants with each other. The couple
say they want to trade hearts as a symbol of love and have created a new religion where
trading hearts is required for married couples. Both individuals are healthy with no medical
problems or history of medical illnesses. Their HLA typings have not yet been performed.
What should the physician do?

A. Perform the surgeries at the couples request


B. Perform the surgeries only if their HLA profiles are a match and the risks are understood
C. Do not perform the surgeries because it is not a medically necessary procedure
D. Refer the couple for family counseling
E. Refer the couple for psychiatric evaluation
141. A 25-year-old man in medical school comes to the physician complaining of occasional
sharp pain in his lateral right knee after runs. He jogs four miles every morning before the
start of class. No physical abnormality can be palpated in the right knee or leg. Reflexes are
2+ bilaterally. The patient says he does not want to be prescribed glucocorticoids because
it might make him gain weight. The physician prescribes naproxen. Subsequently the
patient explains that the reason for his visit is because he is concerned he might have
osteosarcoma which he is currently reading about. The patient requests an X-ray of his
knee. What should the physician’s response be to this patient’s request?

A. “You are unlikely to have osteosarcoma because you are above age 20.”
B. “You are unlikely to have osteosarcoma because no abnormalities are palpable.”
C. “Have you had retinoblastoma in childhood?”
D. “If naproxen does not relieve your pain, we can further evaluate your condition.”
E. “I will give you an X-ray of your knee.”

142. A 13-year-old girl who is a recent immigrant from China comes to the physician for her first
annual physical examination. Her temperature is 36.8 ℃ (98.2 ℉), blood pressure is 148/88,
pulse is 80/min, and respirations are 17/min. Physical examination shows tanner stage one.
She has not yet started menarche. Serum laboratory results reveal low potassium, low
cortisol, and low sex hormones. This patient most likely has an enzymatic defect in which
cellular organelle?

A. Nucleus F. Rough endoplasmic reticulum (RER)


B. Nucleolus G. Smooth endoplasmic reticulum (SER)
C. Mitochondrion H. Plasma membrane
D. Lysosome I. Golgi apparatus
E. Peroxisome J. Cytosol

143. A 33-year-old primigravida woman at 12 weeks gestation comes to the physician for
routine prenatal care. An ultrasound reveals a male fetus. During male fetal development,
the sex-determining region of Y (SRY) gene is activated early in embryogensis. This
process allows the primordial cells of the gonads to differentiate into the Sertoli cells, which
inhibits female sexual development, and the Leydig cells, which promotes male sexual
development. Which male sexual organ most likely developed first in this patient’s fetus?

A. Glans penis
B. Vas deferens
C. Epididymis
D. Testis
E. Scrotum

144. A six-year-old boy is brought to the physician because of a large scrotum that has been
present since birth. Physical examination shows a bilaterally enlarged scrotum. The
scrotum transilluminates with a light source. It is determined that the scrotal sac is filled with
fluid from the peritoneal cavity. What duct like structure abnormally failed to obliterate early
in this patient’s life?

A. Tunica albuginea
B. Tunica vaginalis
C. Processus vaginalis
D. Allantois
E. Paramesonephric duct

145. If the intestines of the patient above were to fall into the scrotal sac through the same
pathway as the fluid, this patient would have what type of hernia?

A. Direct inguinal hernia


B. Indirect inguinal hernia
C. Femoral hernia
D. Umbilical hernia
E. Hiatal hernia

146. A scientist is studying the humpback anglerfish (​M. johnsonii​). Very little is known about the
anglerfish genome or gene expression because it has never been studied before. Ten live
specimens arrive on the first day of research. The scientist notices that one anglerfish has a
large tumor on its jaw. What is the best initial assay to evaluate the similarities and
differences between the tumor cells and normal somatic cells of the humpback anglerfish?

A. Northern blot
B. Southern blot
C. DNA microarray
D. Polymerase chain reaction (PCR)
E. Enzyme-linked immunosorbent assay (ELISA)

147. There are sometimes inconsistencies between textbook scenarios and real life scenarios.
For instance, a patient with a mitochondrial disease is expected to have a mother with the
same disease, but this is often not the case. What phenomenon best explains why a patient
has a mitochondrial disease but his/her mother does not?

A. Variable expression
B. Codominance
C. Incomplete dominance
D. Heteroplasmy
E. Multifactorial inheritance

148. There are sometimes inconsistencies between textbook scenarios and real life scenarios.
For instance, a patient with Huntington’s disease is predicted to have a parent with
Huntington's disease, but this is often not the case. In fact, many Huntington’s patients are
the first in their family to have Huntington’s disease. What most likely accounts for this
textbook vs. real life discrepancy?

A. Missed diagnosis
B. Incomplete penetrance
C. Anticipation
D. Variable expression
E. Lead time bias

149. Four medical students took the United States Medical Licensing Examination (USMLE)
Step One on the same day. They each got their score three weeks after the exam. One
student scored 132, one student scored 272, and two students scored 274. What is the
median score of the four students?

A. 70.67
B. 142
C. 238
D. 273
E. 274

150. A 28-year-old African American woman comes to the physician because of a two day
history of dyspnea. The patient works as the manager of a laundromat. She occasionally
uses FlexoPlex, an over-the-counter natural extract for joint pains. Family history is
unremarkable. She does not use tobacco, alcohol, or illicit drugs. Physical examination
shows bilateral maculopapular rashes above the ankles which the patient reports as
occasionally itchy. Crackles are discovered on auscultation, but sputum culture is negative.
Serum studies indicate an elevation in angiotensin-converting enzyme (ACE) and calcium.
A complete blood count is normal. A pulmonary function test shows a FEV​1​/FVC ratio of
0.81. Chest X-ray reveals bilaterally enlarged hilar lymph nodes. A lung biopsy is taken and
shown below.

​Courtesy of Yale Rosen

What is the most likely diagnosis?


A. Eosinophilic granulomatosis with polyangiitis (EGPA)
B. Advanced autoimmune deficiency syndrome (AIDS)
C. Chronic obstructive pulmonary disease (COPD)
D. ​Pneumococcal pneumonia
E. Sarcoidosis

151. A 17-year-old boy is rushed to the emergency room after collapsing on the river at a
regatta​ while representing his high school rowing team during the race. The patient’s past
medical history is unremarkable, however he does have a family history of sudden death.
He takes no medications and does not use tobacco, alcohol, or illicit drugs. Despite the
best effects to resuscitate the patient, he expires shortly after arrival. An autopsy is
performed and the patient’s heart (right) is compared to a normal heart (left) as shown in
the image below.

This patient’s heart can be best described as having which of the following conditions?

A. Atrophy
B. Hypertrophy
C. Tamponade
D. Amyloidosis
E. Pericarditis

152. In a small Amish community of 2,000 people in Western Pennsylvania, red-green color
blindness is prevalent. The allele frequency of this X-linked recessive disease is 1/10.
There are 1,000 females in this community in which 10 have Turner syndrome. There are
also 1,000 males in this community in which 100 have Klinefelter syndrome. How many
people are likely to have red-green color blindness in this small Amish community?

A. 10
B. 43
C. 87
D. 102
E. 146
153. A 30-year-old primigravida woman at 33 weeks gestation comes to the physician for a
routine prenatal care check-up. The patient feels well and has no complaints. She does not
have any underlying medical conditions and does not take any prescription medications.
She does not use tobacco, alcohol, or illicit drugs. Physical examination shows well
developed breasts consistent with the stage of her pregnancy. Ultrasound of the uterus
shows a normal developing fetus. Serum prolactin levels have increased since her last
prenatal visit one week ago, but lactation has not yet started. Which hormone plays an
important role in breast development, increased serum prolactin, and inhibition of lactation
for this patient?

A. Oxytocin
B. Estrogen
C. Progesterone
D. Luteinizing hormone (LH)
E. Follicular stimulating hormone (FSH)

154. A screening test is developed in a local community as a method of secondary prevention


for breast cancer. Women may choose to have their breasts palpated by a physician at
their annual physical evaluation. If a lump is palpable, a biopsy will be sent to a laboratory
for confirmatory analysis. A sample of 500 patients are analyzed to measure the quality of
the new screening test. The results are shown.

Screening Positive Screening Negative

Biopsy Negative 56 129

Biopsy Positive 143 172

What is the sensitivity of this new breast cancer screening test?

A. 28.1%
B. 42.9%
C. 45.4%
D. 69.7%
E. 71.9%

155. The purified protein derivative (PPD) test is a screening tool for tuberculosis. Tuberculin
antigen is injected under the skin and examined 48-72 hours later. If there is a large
induration, the test is positive. A sample of 500 volunteers took the PPD test and then
underwent a confirmatory sputum culture for tuberculosis. The results of the test are shown.

Sputum Culture + Sputum Culture -

PPD + 172 56
PPD - 129 143

What is the specificity of this screening test?

A. 172 / (172 + 56)


B. 172 / (172 + 129)
C. 143 / (143 + 56)
D. 143 / (143 + 129)
E. (172 + 143) / (172 + 143 + 129 + 56)

156. A 50-year-old man comes to the physician due to easy bruising. The patient has a six-day
history of bruises on his arms and legs but does not recall any trauma. No similar
experiences in the past are reported. The patient was diagnosed with bacterial pneumonia
one month ago that was successfully treated with antibiotics. His family history is
unremarkable. He does not use tobacco, alcohol, or illicit drugs. Physical examination
shows several moderate size bruises on his arms and legs. A mass can be palpated in the
left hypochondrium just below the costal margin. A complete blood count indicates low
lymphocytes, low myelocytes, low erythrocytes, low platelets, and a high CD20+/CD3+
ratio. Needle biopsy of the bone marrow yields little fluid. The biopsy specimen under
microscopy is shown below.

​Courtesy of Paulo Henrique Orlandi Mourão

What is the most likely diagnosis?

A. Hemophilia A
B. Von Willebrand disease
C. T-cell acute lymphoblastic leukemia (T-ALL)
D. Multiple myeloma
E. Hairy cell leukemia

157. A 57-year-old man with type 2 diabetes mellitus comes to the physician due to a painful
lesion on his left scrotum. Metformin and canagliflozin are his only medications.
Temperature is 37.9 ℃ (100.2℉), blood pressure is 120/80 mm Hg, pulse is 80/min, and
respirations are 19/min. Fasting glucose concentration is 107 mg/dL and hemoglobin A1c is
6.7%. An image of the patient’s scrotum is shown below.

What is the most likely cause of this patient’s scrotal lesion?

A. Infection
B. Ischemia
C. Hemorrhage
D. External trauma
E. Ultraviolet radiation

158. A researcher is analyzing familial hypertension in rats. He discovers that several members
of a rat family have hypertension across three generations. The family pedigree is shown
below.

What is the most likely mode of inheritance in this rat family?

A. Autosomal dominant
B. Autosomal recessive
C. X-linked dominant
D. X-linked recessive
E. Holandric
F. Mitochondrial

159. A 14-year-old boy is attending tryouts for his high school cross country team. He has no
previous experience as a competitive runner and has been less active than most
adolescents his age. The boy takes no medications. His family history is unremarkable. He
does not use tobacco, alcohol, or illicit drugs. The coach orders the boy to run five miles
with his teammates. After one mile, the boy stops running. His pulse is 170/min and
respirations are 33/min. Which organ system is the primary limiting factor preventing this
boy from completing his run?

A. The respiratory system


B. The cardiovascular system
C. The renal system
D. The alimentary system
E. Musculoskeletal system

160. A 34-year-old man comes to the physician due to progressive hearing loss that has been
worsening for the past several months. The patient comes from a deeply religious family
that strongly believes in homeopathic medicine. This is his first visit to an allopathic clinic
after being unsatisfied with homeopathic results. The patient reports having hematuria
since childhood. His family history is significant for hematuria and hearing loss. He does not
use tobacco, alcohol, or illicit drugs. Physical examination reveals bilateral sensorineural
hearing loss. The patient’s family pedigree is shown below.

​Courtesy of Madibc68

What is the most likely mode of inheritance in this patient’s family?

A. Autosomal dominant
B. Autosomal recessive
C. X-linked dominant
D. X-linked recessive
E. Holandric
F. Mitochondrial
G. Multifactorial

161. A 71-year-old man comes to the physician because of several instances of transient loss of
vision in his left eye. The vision loss occurs without warning and “falls like a curtain.” After
several seconds, his vision returns. There is no pain during loss of vision. The patient
currently takes no medications. Family history is unremarkable. He has a one pack per day
smoking history for the past 40 years and does not use alcohol or illicit drugs. Temperature
is 37℃ (98.6℉), blood pressure is 145/93, pulse is 90/min, and respirations are 16/min.
Fundoscopic examination is normal and no other abnormalities are observed. Complete
blood count is within reference range. What is the most likely cause of this patient’s
transient loss of vision?
A. Atherosclerosis
B. Retinal detachment
C. Demyelinating plaque
D. Increased red blood cell mass
E. ​Treponema pallidum ​infection
F. Vitamin B​12​ deficiency

162. In a community of 1,000 people, one in 100 individuals are living with cystic fibrosis.
Approximately how many individuals are carriers of a mutated cystic fibrosis
transmembrane conductance regulator (CFTR) gene?

A. 100
B. 180
C. 300
D. 432
E. 632

163. In a community of 1,000 people, 100 individuals are living with cystic fibrosis.
Approximately how many individuals are carriers of a mutated cystic fibrosis
transmembrane conductance regulator (CFTR) gene?

A. 100
B. 180
C. 200
D. 432
E. 632

164. A 21-year-old man is rushed to the hospital after being involved in a head-on motor vehicle
collision with another car while driving on the highway. He is unconscious and has lost a
great deal of blood. A blood transfusion is required for survival. A card is found on the
patient’s key chain as shown below.

An advanced directive is found in the patient’s wallet confirming he is a lifelong Jehovah’s


Witness who does not want blood transfusions under any circumstance. The patient’s
girlfriend then rushes into the hospital and shows the physician a text message from the
patient several minutes ago saying that he intends to kill himself over his failing grades in
college. What should the physician do?

A. Transfuse blood because unconsciousness is implied consent


B. Transfuse blood because suicidality is automatic loss of capacity
C. Give fluids only because the advance directive instructs not to give blood
D. Get a court order to transfuse blood.
E. Call the ethics committee

165. A 69-year-old woman is rushed to the emergency room after being found collapsed in her
house during a fire rescue. She is unconscious on arrival. Pulse is 75/min and respirations
are 16/min. She has on a bracelet that reads “Do Not Resuscitate (DNR).” The patient’s
medical chart confirms the DNR order. A fingertip blood analysis shows a
carboxyhemoglobin to hemoglobin ratio of 30%. What should the physician do?

A. Place the patient in a hyperbaric chamber because DNR does not apply for this
treatment
B. Only give injection drugs but not 100% oxygen which is a type of pulmonary and
cardiovascular intervention
C. Wait for the patient’s family to be contacted to hear their opinion
D. Hakuna matata

166. A 17-year-old boy comes to the physician for his annual required physical examination
before the start of his high school cross country season. The patient tells the physician that
despite intensive training, he is slower than the average runner and is often out of breath at
the end of each race. He takes no medications and does not use tobacco, alcohol, or illicit
drugs. Family history is significant for mild left ventricular hypertrophy later in life. Physical
examination shows a 175 cm (5’9) male with lean muscle, BMI 20 kg/m​2​. A systolic murmur
can be heard at the second right intercostal space near the sternal border. If this patient’s
condition is hereditary, he most likely has a mutation in which gene?

A. ​TTN
B. ​FBN1
C. ​NOTCH1
D. ​BMPR2
E. ​MYH7

167. A 34-year-old woman comes to the physician because of bleeding gums and easy bruising.
She is a hematologist at the same hospital as the physician. The patient says her
symptoms began a week ago without warning but she doesn't know why. Physical
examination confirms her complaints. Laboratory results are shown below.

Hematologic:
Hemoglobin: 13 g/dL
Hematocrit: 38%
Platelet count: 200,000/mm​3
Plasma:
Prothrombin time (PT): 20 sec
Partial thromboplastin time (PTT): 37 sec
Bleeding time: 6 min

Fresh frozen plasma is started and her symptoms resolve. What is the most likely
diagnosis?

A. Von Willebrand disease


B. Glanzmann’s thrombasthenia
C. Immune thrombocytic purpura
D. Thrombotic thrombocytopenic purpura
E. Factitious disorder

168. A 30-year-old man comes to the physician because of frequent daytime sleepiness
accompanied by cranky moods. The patient is a Sumo wrestler who has recently found it
difficult to concentrate during practice and tournaments. He was diagnosed with
Prader-Willi syndrome as a child and has tried to lose weight until he decided to enroll in
wrestling at the age of 15. Blood pressure is 140/90 mm Hg. He is 183 cm (6 feet) and
weighs 226 kg (500 lbs), BMI 67.8 kg/m​2​. Laboratory results show evidence of prediabetes.
The physician recommends a weight loss regimen, but the patient says “I can’t doctor, I am
a Sumo wrestler and I am required to maintain a heavy weight.” This patient’s choice of
career is most likely an example of which of the following defense mechanisms?

A. Denial
B. Fixation
C. Sublimation
D. Suppression
E. Rationalization
F. Intellectualization

169. A scientist is engineering a virus for bioterrorism. Which cell type is the best target for the
virus in order to avoid a host immune response?

A. Myocytes E. Neurons
B. Erythrocytes F. Parietal cells
C. Melanocytes G. Epithelial cells
D. Osteoblasts H. Endothelial cells

170. An 18-year-old woman comes to the physician due to progressive skin wrinkling. The
patient was diagnosed with Warkany syndrome 2 (trisomy 8) at birth. Physical examination
shows skin atrophy, pinpoint bleeding, and a pale complexion. If the vast majority of
individuals with Warkany syndrome 2 die in utero, and the majority of live births die before
adulthood, this patient most likely has which of the following additional underlying
conditions?

A. Albinism
B. Progeria
C. Increased bleeding time
D. Mosaicism
E. A robertsonian translocation

171. A two-year-old girl is brought to the physician by her parents due to painful blistering skin
when out in daylight for short periods. The family lives on a Cherokee Indian reservation
that prefers traditional naturopathic medicine over allopathic medicine. The patient is
visiting an allopathic physician for the first time because her parents feel her symptoms are
too severe for their tribal medicine man. The patient does not take any prescription
medications. Physical examination shows many freckles, rough scaly skin with scattered
hyperpigmentation, telangiectasias, and bloodshot eyes with corneal ulcerations. The
blisters are warm, swollen, and blanch with pressure. Laboratory results are normal and
sputum culture is negative. What is the most likely cause of this patient’s blisters?

A. Dermatitis herpetiformis
B. Erythema multiforme
C. Steven-Johnson syndrome
D. Superficial burn (1°)
E. Superficial partial thickness burn (2°)

172. A 40-year-old Asian man comes to the physician after a fainting spell when lifting a heavy
box overhead onto a shelf. The patient has never experienced similar symptoms in the
past. He currently takes no medications and does not use tobacco, alcohol, or illicit drugs.
His father died suddenly at age 43, but an autopsy was unable to determine the cause of
death. The patient’s pulse is 60/min and blood pressure is 118/79 mm Hg.
Electrocardiogram (ECG) shows ST segment elevation in leads V​1​-V​3​. Chest X-ray reveals
no abnormalities. Serum troponin is negative. What is the most likely diagnosis?

A. Dilated cardiomyopathy
B. Hypertrophic cardiomyopathy
C. Brugada syndrome
D. Wolff-Parkinson-White syndrome
E. Myocardial infarction (MI)

173. A 19-month-old girl is brought to the physician after swallowing some of her father’s
erythromycin prescription medication. The drug most likely causes which of the following
effects in this child?
A. Mydriasis
B. Sedation
C. Peristalsis
D. Paralysis
E. Hydrophobia

174. An 18-year-old man comes to the physician due to several pus-leaking sores and
honey-crusted scabs on his face. The patient has never seen a physician previously
because his parents are strong believers in naturopathy. He has had various infections in
the past treated with home remedies. The patient does not use tobacco, alcohol, or illicit
drugs. His temperature is 37.8℃ (100.0℉). In addition to the facial findings, patches of red
thickened skin are noted on the hands and wrists bilaterally. A large bruise is seen on the
right elbow. Laboratory results show the following.

Hematologic:
Hemoglobin: 15 g/dL
Hematocrit: 48%
Leukocytes: 6,000/mm​3
CD4+ T-cell: 800/mm​3
Platelets: 100,000/mm​3
Serum:
IgA: 600 mg/dL
IgE: 600 IU/mL
IgG: 680 mg/dL
IgM: 45 mg/dL

What is the most likely genetically based diagnosis for this patient?

A. DiGeorge syndrome
B. Severe combined immunodeficiency syndrome (SCID)
C. Ataxia-telangiectasia
D. Hyper-IgM syndrome
E. Wiskott-Aldrich syndrome

175. A 14-year-old girl is brought to the physician with an acute asthma attack. The patient was
hiking in the woods with a friend when she fell short of breath and realized she forgot her
inhaler at home. Wheezing can be heard on auscultation. Arterial blood gas analysis
reveals the following values.

pH: 7.25
PaCO​2​: 55 mm Hg
HCO​3​-​: 24 mEq/L

This patient’s pH is most likely decreased through which of the following mechanisms?
A. Carbon dioxide and water reaction
B. Bicarbonate and chloride exchange
C. Carbon dioxide and bicarbonate reaction
D. Decreased hydrogen excretion in the collecting ducts
E. Carbon dioxide binding hemoglobin to form carbaminohemoglobin

176. A 76-year-old woman comes to the office as a new patient with concerns of back pain. The
physician is 30 minutes late and apologizes on arrival. The patient smiles kindly and says,
“There is no need to explain. I know you are a good doctor because I choose my caregivers
based on their political affiliation as indicated on the voter registration database. That way I
know which doctors have my best interest in mind.” During a brief discussion with the
patient, she adds that her favorite part of election debates is the “show of hands
responses,” where the host makes a statement and all candidates who agree raise their
hands. This patient is most likely using which defense mechanism?

A. Altruism
B. Reaction formation
C. Rationalization
D. Splitting
E. Identification

177. An eight-year-old boy is brought to the physician due to fatigue and unintentional weight
loss. The patient has lost 5 kg (11 lbs) despite adequate nutrition. Physical examination
shows a large bruise on the left forearm with no explainable reason. The liver can be
palpated four centimeters below the right costal margin. A complete blood count reveals
increased lymphoblasts. A subsequent blood smear confirms acute lymphoblastic
leukemia. Asparaginase is administered through intramuscular injection. This drug
counteracts which tumor enzyme within the patient’s cancer cells?

A. Serine/threonine kinase
B. Tyrosine kinase
C. GTPase
D. Asparagine synthase
E. Telomerase

178. A 53-year-old man comes to the physician due to a five-day history of lesions on his face.
The patient is a recent refugee from a war stricken impoverished country. He cannot
provide accurate information on his past medical history or family history. Physical
examination shows several lesions of diffusely thickened skin on the forehead, chin, ears,
and nose. The lesions are insensitive to touch. Decreased sensation is noted on the
fingertips and toes bilaterally. An immunoglobulin blood test shows high serum antibody
levels. Skin biopsy stained with Ziehl-Neelsen reveals acid-fast bacilli. Which drug
combination therapy will most likely ease this patient’s symptoms?
A. ​Trimethoprim / sulfamethoxazole (TMP/SMX)
B. Amoxicillin / clavulanic acid
C. Rifampin / isoniazid / pyrazinamide / ethambutol / streptomycin
D. Dapsone / rifampin / clofazimine
E. Ceftriaxone / doxycycline

179. A 49-year-old woman comes to the physician due to a two-day history of numerous lesions
on her body. The patient first noticed the lesions on her oral mucosa, but the lesions have
since spread in a synchronous fashion from head to toe. Past medical history is negative
for vaccinations. She works as a time traveler employed by God to moderate history.
Physical examination shows vesicular lesions across the entire body, but are most
concentrated on the face and limbs. All vesicles appear to be in the same stage of
development. What is the most likely diagnosis?

A. Chickenpox
B. Smallpox
C. Measles
D. Mumps
E. Rubella
F. Herpes simplex

180. A 55-year-old man comes to the physician with a left arm cramp after trying to stop a
punching bag with his left hand 30 minutes ago. He has a one-pack-day smoking history for
the past 25 years and uses anabolic steroids for sports performance. The patient’s arm is
shown below.

Significant pain is produced when the patient is asked to pronate or supinate his hand.
Damage to which anatomical structure is the most likely cause of this patient’s symptoms?

A. Annular ligament
B. Biceps tendon
C. Interosseous membrane
D. Radial collateral ligament
E. Ulnar collateral ligament

181. A 39-year-old man comes to the physician due to a two-week history of pain in his right
elbow when pitching. The patient is a major league baseball player who has been pitching
since the age of five. He reports the pain as most severe on the acceleration phase of his
throws. Pain can be reproduced when the fist is clenched. Physical examination shows
moderate swelling on the medial aspect of his right elbow. This patient is most likely
experiencing damage to which anatomical structure?

A. Annular ligament
B. Biceps tendon
C. Interosseous membrane
D. Radial collateral ligament
E. Ulnar collateral ligament

182. A 66-year-old woman with HIV comes to the physician due pelvic pain and an inability to
control urination. The patient has a history of glioblastoma multiforme in which she has
undergone multiple surgeries requiring urinary catheterization. Her temperature is 38℃
(100.4℉), blood pressure is 120/80 mm Hg, and pulse is 100/min. Suprapubic tenderness is
observed on palpation. Urine cultures grow pink colonies on MacConkey agar with no
intermediate white colony phase. Microscopic examination shows gram-negative rods. The
indole test is negative. This patient most likely has an infection caused by which of the
following organisms?

A. ​Enterobacter cloacae
B. ​Escherichia coli
C. ​Serratia marcescens
D. ​Pseudomonas aeruginosa
E. ​Proteus mirabilis

183. A 35-year-old man comes to the physician due to severe itching in his groin that has
increased in intensity for the past two weeks. The patient works as a pimp at a nightclub
and describes himself as a “𝛂 male.” He admits to having numerous sexual partners for the
past several months, but uses condoms infrequently. Physical examination shows
abundant pubic hair. No other abnormalities are observed. A sample of the pubic hair is
shown below under light microscopy.
What is the most appropriate pharmacotherapy for this patient’s illness?

A. Penicillin
B. Permethrin
C. Pyrantel pamoate
D. Praziquantel
E. Albendazole
F. Metronidazole

184. “Old habits die hard” is an idiom that means people find it difficult to change activities they
have been doing for a long time. A smoker will smoke again. An alcoholic will drink again. A
drug addict will illicit drugs again. An obese person will overeat again. A murderer will kill
again. A rapist will rape again. Which of the following factors is the strongest predictor for
completed suicide in a patient?

A. Access to firearm F. History of childhood trauma


B. Advanced age G. History of suicide attempt
C. Male sex H. Family history of suicide
D. Female sex I. Recent marital separation
E. Depression J. Financial difficulties

185. A 46-year-old woman comes to the physician due to a two-week history of dyspnea and
dysphagia. The patient takes no medications and does not use tobacco, alcohol, or illicit
drugs. Her temperature is 37℃ (98.6℉), blood pressure is 117/70 mm Hg, pulse is 60/min,
and respirations are 25/min. Jaundice and scleral icterus are noted. A large, firm,
nontender, irregular mass can be palpated in the center of the anterior neck. Serum studies
show elevated immunoglobulin gamma 4 (IgG​4​). A biopsy of the neck mass is shown
below.
​https://step1.medbullets.com/step1-endocrine/109074/riedel-thyroiditis

Prednisone is prescribed and her symptoms resolve. What is the most likely thyroid
abnormality in this patient?

A. Chronic autoimmune thyroiditis (Hashimoto)


B. Toxic diffuse goiter (Graves’)
C. Subacute granulomatous thyroiditis (de Quervain)
D. Riedel thyroiditis
E. Anaplastic thyroid carcinoma

186. ​A 26-year-old man comes to the physician due to sharp back pain that started five minutes
ago. His blood pressure is 90/60 mm Hg, pulse 180/min, respirations 31/min. Physical
examination shows a 6’6 (198 cm) male with long fingers and a compressed chest. The
lenses of his eyes appear to be misaligned with his pupils. Despite life saving measures,
the patient expires. On autopsy, a ruptured aorta and a prolapsed mitral valve are
discovered. What is the most likely cause of this patient’s death?

A. Degradation of elastin by elastase


B. A defect in a glycoprotein that forms a sheath around elastin
C. A defect in non hydroxylated lysine residue cross-linking
D. Buildup of atherosclerotic plaques that occlude distal arteries
E. Rupture of papillary muscles in the left ventricle

187. A scientist is told that ​E. coli​ ferments lactose. To test this out, he places 100 mg of lactose
into a 10 mL glucose rich broth containing ​E. coli.​ Almost no lactose fermentation is
observed. What accounts for the lack of adequate lactose fermentation in this experiment?

A. Inhibition of adenylate cyclase by glucose


B. Induced expression of catabolite activator protein (CAP) by cyclic AMP
C. Promoter methylation inhibits DNA binding of catabolite activator protein (CAP)
D. Repressor protein is bound to the operator
E. Inadequate amounts of lactose was added to the broth
188. A 53-year-old man visits his physician because of nausea and vomiting that has persisted
for the past two days. His past medical history is insignificant, but he has a brother who
visits the hospital infrequently for sporadic episodes of diarrhea and fatigue. Physical
examination shows a 5’11 (180.34 cm) caucasian male with dark tan skin. No other
physical abnormalities are noted. A blood test shows serum transferrin saturation at 51%
(normal <45%). Further inquiry reveals the patient takes vitamin pills. Which vitamin
supplementation is the patient most likely taking?

A. Retinol (A)
B. ​Cobalamin (B​12​)
C. Ascorbic acid (C)
D. Ergocalciferol (D​2​)
E. ​α-Tocopherol (​E)

189. An 18-year-old man is brought to the physician due to a sudden intense back pain that
started seven minutes ago. His blood pressure is 125/81 mm Hg, pulse 70/min, respirations
14/min. A midsystolic click can be heard at the apex. Physical examination shows a 6’6
(198 cm) male with long fingers and a compressed chest. The lenses of his eyes appear to
be misaligned with his pupils. Despite life saving measures, the patient expires. What is the
most likely cause of this patient’s death?

A. Ruptured berry aneurysm


B. Ruptured papillary muscles in the left ventricle
C. Ruptured aortic aneurysm
D. Aortic dissection
E. Pancoast tumor

190. A five-year-old boy is brought to the physician because he feels like “there is something up
my nose.” He recently immigrated from Cambodia with his family. His parents were farmers
who frequently used human feces as fertilizer for their crops. His temperature is 99.7 ​°F
(37.6 °C), blood pressure is 104/65 mm Hg, and pulse is 95/min. No physical abnormalities
are observed. Using a pair of tweezers, the physician pulls out a 10 inch long organism
from the right nostril. The organism is shown.

​Courtesy of Javier Palaus Soler / Ostman agency


What is the most likely identity of the organism in this patient?

A. Coronavirus
B. ​Enterobius vermicularis
C. ​Trichuris trichiura
D. ​Ascaris lumbricoides
E. ​Toxocara canis

191. A 31-year-old man comes to the physician because of nausea and vomiting that has
persisted for the past 10 days with recent onset of muscle pains. He is a hunter in the Idaho
wilderness and frequently eats exotic game. His temperature is 38.1 °C (100.6 °F), blood
pressure is 123/81 mm Hg, and pulse is 89/min. Periorbital edema is noted around the right
eye. A muscle biopsy taken is shown.

Which medication is most appropriate to treat this patient’s condition?

A. Albendazole
B. Ivermectin
C. Pyrantel pamoate
D. Praziquantel
E. Permethrin cream

192. ​A 58-year-old woman comes to the physician because of a recent appearance of pink
papules on her back and buttocks. She has switched to a vegetarian diet for the past four
months because she started having upper abdominal pain after high fat meals. Physical
examination identifies the papules as xanthomas. A mass can be palpated below the left
costal margin. A tube of blood is drawn from the patient and left to stand overnight. The
results are shown below as compared to a normal blood sample.
​Normal (left), Patient (right)

Which of the following enzymes is likely defective in this patient?

A. HMG-CoA synthase
B. HMG-CoA reductase
C. HMG-CoA lyase
D. ATP Citrate lyase
E. Lipoprotein lipase

193. A 36-year-old woman gravida 1, para 0, visits her gynecologist after a positive home
pregnancy test. Ultrasound confirms a fetus at approximately 10 weeks gestation. Due to
abnormalities detected on the ultrasound, a transabdominal chorionic villus sample (CVS)
is extracted and sent for cytogenetic analysis. The result of the karyotype is 47,XY,+16 as
shown.

​https://kateandaly.wordpress.com/tag/trisomy-16/

What is the most likely fate of the fetus?

A. Spontaneous abortion (miscarriage)


B. Failure to thrive
C. Severe intellectual disability
D. Short adult height
E. Early dementia
194. A 33-year-old male immigrant is brought to the physician by his wife because of a five day
history of hallucinations. His temperature is 36.7 °C (98.2 °F), blood pressure is 101/60 mm
Hg, and pulse is 102/min. He is thin. A circle of darkened skin is seen around his neck at
the C3/C4 dermatome. A drug with similar properties to which vitamin might relieve this
patient’s symptoms?

A. Thiamine (B​1​)
B. Niacin (B​3​)
C. Pentothenic acid (B​5​)
D. Biotin (B​7​)
E. Folate (B​9​)
F. Cobalamin (B​12​)

195. For several years it has been noted that children living in one small town have had an
increased incidence of Non-Hodgkin’s lymphoma compared to the national average. It is
suspected that a nearby nuclear power plant is a risk factor. A study is undertaken to
determine the relative risk of developing lymphoma for children living near the power plant.
A sample of 1,000 school age children from this town are selected and followed for five
years. This sample is compared to a sample of 1,000 children from a neighboring town also
followed for five years during the same period. It is discovered that 120 children living near
the power plant developed lymphoma and 20 children in the neighboring town developed
lymphoma. Approximately how many children must live near the power plant for one child
to be adversely affected?

A. 6
B. 10
C. 20
D. 100
E. 120

196. A researcher is studying the effects of low oxygen tension on the liver. She puts a colony of
mice in a hypobaric chamber with 200 millibars of atmospheric air for 24 hours. The mice
are subsequently dissected and their livers biopsied. Which histological feature of the
hepatocytes in these mice suggests irreversible cellular injury?

A. Lysosomal membrane rupture


B. Ribosomal detachment from the endoplasmic reticulum
C. Mitochondrial swelling
D. Chromatin clumping
E. Plasma membrane blebbing

197. A 64-year-old man comes to the physician because of a sudden onset of dyspnea and
fatigue that started 30 minutes ago. He says he was home reading a newspaper when he
suddenly felt dizzy. Past medical history is significant for a myocardial infarction 22 months
ago. His current medications include metoprolol, hydrochlorothiazide, atorvastatin,
levodopa, and carbidopa. Physical examination shows pitting edema in the legs and feet.
Upon questioning, he reveals that he has not been taking his medications due to financial
stress, but promises to continue to do so from now on. Which drug should be removed and
replaced for this patient?

A. Metoprolol
B. Hydrochlorothiazide
C. Atorvastatin
D. Levodopa
E. Carbidopa

198. A local government has introduced a new screening test for the coronavirus disease of
2019 (COVID-19) as a method of secondary prevention. One hundred adults visiting their
physicians due to shortness of breath are screened by looking for evidence of pneumonia.
A throat swab is taken of all 100 patients and sent to a laboratory to be tested for
COVID-19 nucleic acid. The results are shown.

RT-PCR Positive RT-PCR Negative

Screening positive 94 2

Screening negative 2 2

Which statement is most accurate about the screening test?

A. It has a positive likelihood ratio > 10 indicating high sensitivity


B. It has a positive likelihood ratio > 10 indicating high specificity
C. It has a negative likelihood ratio < 0.1 indicating high sensitivity
D. It has a negative likelihood ratio < 0.1 indicating high specificity

199. A 16-week-old girl is brought to the pediatrician because of frequent vomiting after almost
every meal. The pregnancy was uncomplicated at term, but polyhydramnios was noted
prior to birth. The child weighs 5.8 kg (25th percentile). Physical examination reveals no
abnormalities and bile is absent from the vomit. An abdominal X-ray is shown.
What is the most likely cause of vomiting in this patient?

A. Esophageal atresia
B. Pyloric stenosis
C. Annular pancreas
D. Duodenal atresia
E. Hirschsprung’s disease

200. A 12-year-old girl is brought to the pediatrician due to failure to thrive. She is at the 15th
percentile for height and 70% percentile for weight. Physical examination shows round
puffy cheeks and shortened 4th and 5th fingers compared to the other digits. Tapping her
cheeks with a pen results in contraction of her facial muscles. Maternal family history is
significant for a ​GNAS1​ gene mutation that causes defective bone and kidney response to
parathyroid hormone (PTH). What are the expected serum laboratory results for this patient
based on family history and clinical findings?

Parathyroid Calcium (Ca​2+​) Phosphate (PO​43-​


​ )
hormone (PTH)

A. ↓ ↓ ↔

B. ↓ ↔ ↔

C. ↑ ↑ ↓

D. ↑ ↓ ↑

E. ↓ ↑ ↓

201. Molar pregnancies are relatively common in the United States, occurring in one out of
every 1,000 pregnancies. When a mole contains ⅔ DNA from the mother and ⅓ DNA from
the father, it results in a large fetus with a small placenta. Conversely, if the mole contains
⅓ DNA from the mother and ⅔ DNA from the father, it results in a small fetus with a large
placenta. Given this information, what is the likely outcome of a fertilized ovum that only
contains DNA of paternal origin?

A. Large fetus with no placenta


B. Large placenta with no fetus
C. Normal fetus with normal placenta
D. Malformation of fetal structures due to genetic imprinting
E. Development of choriocarcinoma

202. A 15-year-old boy comes to the physician because of severe pain in his right thigh. The
pain began two weeks ago which he attributed to growing pains, but has since gotten
worse. X-ray of the femur shows a mass on the lateral border of the femoral shaft. A
histological biopsy of the mass reveals small-blue-round-cells on light microscopy.
Subsequent cytogenetic analysis uncovers an 11;22 chromosomal translocation confirming
Ewing’s sarcoma. The boy does not appear surprised by the diagnosis and shows little
emotion. He responds by demonstrating some knowledge of clinical laboratory science. He
later asks if Ewing’s sarcoma might arise from the same physiologic mechanisms as a
growth spurt, as both involve proliferation of somatic cells. Which defense mechanism is
this patient most likely exhibiting?

A. Dissociation
B. Intellectualization
C. Isolation of affect
D. Rationalization
E. Reaction formation

203. A three-day-old male infant is brought to the physician because of a large production of
bilious vomitus an hour previous. The pregnancy was uncomplicated and prenatal
cytogenetic analysis had shown a normal karyotype. He has failed to pass stool since birth
and physical examination shows a distended abdomen. A rectal suction biopsy observed
under light microscopy found no ganglionic cells. If this defect is caused by a failure of
neural crest cell migration, what other abnormality has an increased likelihood of being
present in this patient?

A. Single palmar crease


B. Intellectual disability
C. Incurved 5th finger
D. Atrioventricular septal defect
E. Acute lymphoblastic leukemia (ALL)

204. A 25-year-old pregnant woman gravida 1, para 0, comes to the gynecologist due to recent
episodes of vomiting and irregular vaginal bleeding. She is concerned that her symptoms
are the initial signs of an aborting fetus. Ultrasound shows a grape-like mass in the uterus
with no apparent fetus. A biopsy of the mass confirms a molar pregnancy. The mass is
evacuated using uterine suction and sent for cytogenetic testing. The analysis reveals a
46,XY karyotype. The abnormal mass in this patient is most likely the result of which
pathophysiologic mechanism of fertilization?

A. Two sperm + one egg


B. Two sperm + enucleated egg
C. One sperm + diploid egg
D. One sperm + enucleated egg (subsequently duplicates paternal DNA)
E. One sperm + one egg

205. A 23-year-old woman at 24 weeks gestation, gravida 1, para 0, visits her physician for
routine prenatal care. Previous blood typing identified her as rhesus factor negative (Rh​-​),
and her male partner as rhesus factor positive (Rh​+​). Patient serum ELISA detected
insignificant amounts of anti-D IgG titers. She is given an intramuscular injection of
RhoGAM to prevent hemolytic disease of the newborn in subsequent pregnancies, an
immune mediated disease. If RhoGAM is an antibody against rhesus factor, how does this
drug suppress the maternal immune system?

A. Blocks maternal preformed IgG immunoglobulins from crossing the placenta


B. Binds maternal IgM immunoglobulins
C. Induce maternal B cell apoptosis by binding B cell Fas receptors
D. Enters maternal B cell nuclei and alter transcription of immunoglobulin genes
E. Competes with maternal IgM for fetal red blood cells in maternal circulation

206. A seven-year-old-boy is brought to the physician with a large mass at the angle of his right
jaw. He is a recent immigrant from Nigeria and his past medical history is unclear. Physical
examination shows a large ulcerating tumor with pus drainage. Histology from a biopsy
confirms the tumor is of B cell origin. The physician suspects a translocation between
chromosome 8, containing the proto-oncogene ​c-myc​, with chromosome 14, containing a
gene coding for immunoglobulin heavy chain. However, karyotype results instead reveal an
8;22 translocation. The translocation in this patient most likely involves ​c-myc​ and a gene
that encodes for which protein?

A. Tumor protein 53 (TP53)


B. Neurofibromin 2
C. Lambda light chain
D. ​Breast cancer type 1 susceptibility protein (BRCA1)
E. ​RNA-binding protein EWS

207. ​An 87-year-old woman comes to the physician because of difficulty swallowing solid food.
Her dysphagia has been getting worse over a period of months. She also reports feeling
fatigued, which she attributes to her increasing age. Her stool is occasionally bright red,
although usually her stool is a near black color. Physical examination shows cracks on the
corners of the mouth, conjunctival pallor, and caved in nails. A barium swallow test reveals
strong evidence of esophageal webs. Serum laboratory studies are pending. What are the
expected serum laboratory results for this patient?

Serum Iron Transferrin & Total Iron Ferritin % Transferrin Saturation


Binding Capacity (TIBC) (Serum Iron/TIBC)

A. ↓ ↑ ↓ ↓↓

B. ↓ ↓ ↑ ↔/↓

C. ↑ ↓ ↑ ↑↑

D. ↔ ↑ ↔ ↓

208. ​A 25-year-old medical student is referred to the physician by her medical school dean
because of uncharacteristic behaviors. She has been on clinical rotations for the past 14
weeks and has started a new rotation two weeks ago. The student says that for the past
week she has been mixing up patient charts and making simple math errors. When the
resident calls on her, she cannot recall simple knowledge and tends to forget simple
requests. She tells the physician that the new rotation has required a change in her daily
routine. Now she must wake up at four in the morning to get ready and does not get home
until 11 at night. Physical examination shows no abnormalities. What is the most likely
diagnosis?

A. Burn out
B. Fatigue
C. Stress
D. Attention deficit disorder (ADD)

209. ​A 71-year-old man with end-stage renal disease comes to the physician for a scheduled
kidney transplant operation. His monozygotic twin brother agreed to be the donor and the
HLA typing is an exceptionally good match. During the operation, a five inch long mass is
placed in the recipients left iliac fossa. Two days later the patient returns to the hospital with
signs of malignant hypertension, heart failure, peripheral edema, azotemia, uremia,
hypocalcemia, hyperkalemia, hyperphosphatemia, anemia, and metabolic acidosis. The
patient subsequently expires. A five inch long iPhone is recovered from the body at
autopsy. A root cause analysis discovers a five inch long kidney was left on the operating
table after the surgery. The physician made what type of medical error?

A. Active error
B. Latent error
C. Never event

210. ​A 12-year-old boy is brought to the pediatrician because of pain in his shoulder, mid back,
and abdomen that has persisted for the past three days. A mass can be palpated 3 cm
(1.18 inch) below the left costal margin. A peripheral blood smear shows normal and sickle
shaped red blood cells on histology. His parents agree to have him receive a bone marrow
transplant in an attempt to save his spleen. A drug is used to ablate the patient’s bone
marrow prior to the transplant. This drug has which of the following effects on the human
body?

A. Ototoxicity
B. Peripheral neuropathy
C. Myelosuppression
D. Nephropathy
E. Herpes zoster reactivation

211. ​A six-month-old boy is brought to the pediatrician because of persistent intermittent


coughing spells and diarrhea. Family history reveals an older sister who died of severe
combined immunodeficiency (SCID) in infancy. Physical examination shows white plaques
on the tongue and at the back of the pharynx that can be easily scraped away.
Hyperinflated lungs are seen on chest X-ray. If the parents decide to have another child,
what are the chances that a male child will have the same disease?

A. ⅛
B. ¼
C. ⅓
D. ½
E. ⅔

212. ​A 12-year-old girl is brought to the physician by her father because of difficulty in weight
management. The father says that his daughter has recently been homeschooled because
of difficulty in keeping up with classes. The girl says homeschooling is a relief from being
made fun of for being in special education at school. Physical examination shows a girl at
50th percentile in height and 98th percentile in weight. Poor muscle tone is noted.
Cytogenetic studies reveal no abnormalities. What is the most likely cause of this child’s
condition?

A) Deletion in maternal chromosome 15


B) Deletion in paternal chromosome 15
C) Methylation of maternal chromosome 15
D) Methylation of paternal chromosome 15
E) Increased stress from school workload.

213. ​An 85-year-old man comes to the physician because of difficulty sleeping. He has been
waking up more frequently at night for the past three months and he no longer feels
recharged in the morning. Physical examination reveals red eyes which the patient
attributes to a lack of sleep. The patient wants to improve his rest but is concerned about
the possibility of becoming addicted to sleeping pills. The patient has a pet eevee. What is
the most appropriate medication to give this patient?
A. Leafeon D. Umbreon G. Glaceon
B. Vaporeon E. Espeon H. Jolteon
C. Ramelteon F. Flareon I. Sylveon

214. ​The town of Willimantic, Connecticut has a population 18,000 residents. In a period of 20
years, 900 develop breast cancer in which 400 die from the disease. A total of 1,200 deaths
occurred in this time frame. What is the case-fatality rate of breast cancer in this town?

A. 1,200/18,000
B. 400/18,000
C. 400/900
D. 400/1,200
E. 20

215. A 21-year-old man comes to the physician after failing a visual acuity test at the
department of motor vehicles. The patient says he had never needed glasses, but recently
his vision has been blurry when out on errands. He is a medical student who spends many
hours reading books and studying on the computer, yet these activities are unaffected. The
patient takes no medications and his past medical history is unremarkable. Family history is
significant for a maternal grandmother with type II diabetes mellitus and a maternal
grandfather with idiopathic pulmonary fibrosis. The patient does not use tobacco, alcohol,
or illicit drugs. The pupils are equal, round, and reactive to light and accommodation.
Standing 20 feet away from a Snellen chart, the patient’s visual acuity is determined to be
20/50 bilaterally. Fasting blood glucose is 80 mg/dL. What physical abnormality of the eye
is expected to be causing this patient’s current symptom?

A. Increased axial diameter


B. Decreased axial diameter
C. Decreased lens elasticity
D. Opacification of the lens
E. Abnormal curvature of the cornea

216. A 34-year-old single woman with familial hypercholesterolemia visits her genetic counselor
for advice on dating and family planning. She wants to meet the right man on an online
dating website and start a family with him. Her family history is significant for familial
hypercholesterolemia in her father and grandfather. Her serum cholesterol level is 300
mg/dL. If the heterozygous frequency of familial hypercholesterolemia is 1/500 in the
general population, what is the probability that this patient will have a child with a serum
cholesterol level of ≥700 mg/dL?

A. 1/1
B. 1/2
C. 1/4
D. 1/16
E. 1/2000

217. A 33-year-old woman visits the physician because she is concerned that her children will
develop ​Friedreich's ataxia (FA)​. The patient has a two-year-old adopted son and a
newborn biological daughter with her husband. She has a biological brother with FA (not
First Aid) and her husband has a biological sister with FA. Neither of the parents know their
own status as carriers. FA has a 1/40,000 prevalence in the general population. The
disease has complete penetrance. What is the chance that both of this patient’s children
will develop symptoms of FA?

A. 1/2
B. 1/4
C. 1/8
D. 1/16
E. 1/360,000

218. A 20-year-old primigravida woman comes to the physician due to concern over a positive
home pregnancy test. The patient has a history of hematuria and proteinuria in childhood
treated with a kidney transplant. She has developed cataracts and progressive hearing loss
since the age of 17. These symptoms are similarly experienced by the patient’s father and
sister. The patient works as a prostitute to pay for college and to feed her newborn stepson
who she won custody over from her ex-boyfriend. She has unprotected intercourse with
some clients on weekends and speculates that one of them could be the father of her
current pregnancy. If the prevalence of this patient’s disease is 1/10,000 in the general
population, what is the probability that her stepson and her unborn child will both have the
disease?

A. 1/2
B. 1/5000
C. 1/10000
D. 1/20000
E. 1/40000

219. A 60-year-old man comes to the physician due to a one month history of fatigue, general
joint pains, and skin lesions. The patient takes ledipasvir, sofosbuvir, and grazoprevir for
chronic hepatitis C. His temperature is 37℃ (98.6℉), blood pressure is 128/81 mm Hg, and
pulse is 80/min. Physical examination shows multiple raised reddish purple spots on the
forearm and thighs that do not blanch with pressure. Sensation on the fingertips and toes
are decreased. A blood draw is hyperviscous. Clumps are noted at the bottom of the
heparin tube containing the patients blood one hour after standing. Serum laboratory
results are shown below.
​https://medicine.tamu.edu/class-files/webpath16/hemehtml/heme065.htm

What is the most likely diagnosis?

A. Diabetic neuropathy
B. Polycythemia vera
C. Mixed cryoglobulinemia
D. Guillain-Barre syndrome
E. Tuberculoid leprosy

220. An investigator is examining the relationship between bowling and lung cancer. The
investigator discovers that bowlers are more likely to develop lung cancer compared to
nonbowlers. She also discovers that bowlers are more likely to be smokers.

Smoking is an example of a(n) ___________.

A. Confounding variable
B. Effect modification

221. An investigator is examining the relationship between bowling and lung cancer. The
investigator discovers that bowlers who smoke are more likely to develop lung cancer. She
also discovers that nonbowlers who smoke are not more likely to develop lung cancer.

Smoking is an example of a(n) ___________.

A. Confounding variable
B. Effect modification

222. A 64-year-old woman comes to the physician due to progressive difficulty with balance that
has resulted in frequent falls. The patient takes no medications and does not use tobacco,
alcohol or illicit drugs. Her family history is unremarkable. The patient is an animal rights
activist and has not consumed animal products for the past 25 years. Her blood pressure is
116/77 mm Hg and pulse is 69/min. Physical examination shows a large, dark red tongue.
The Romberg sign is positive. The babinski sign is positive bilaterally and grip strength is
decreased bilaterally. What is the most likely diagnosis?
A. Amyotrophic lateral sclerosis
B. Friedreich ataxia
C. Anterior spinal artery occlusion
D. Brown-Séquard syndrome
E. Subacute combined degeneration

223. A 26-year-old woman comes to the physician because of a seven-day history of fatigue
and headaches. The patient recently returned home from a hiking trip in Connecticut two
weeks ago. Her temperature is 37.9℃ (100.2℉), blood pressure is 118/80 mm Hg, and
pulse is 65/min. A rash is discovered on the medial aspect of the patient’s right arm as
shown in the image below.

No other physical abnormalities are observed. This patient most likely came in contact with
which of the following animals?

A. Mouse E. Racoon I. Armadillo M. Cat Q. Black widow spider


B. Dog F. Fox J. Mosquito N. Tsetse fly R. Kissing bug
C. Deer G. Snake K. Tick O. Pufferfish S. Louse
D. Bat H. Snail L. Sandfly P. Human T. Frog

224. A 37-year-old man comes to the physician due to persistent eczema. Physical examination
shows no abnormalities. After the exam, the patient asks if he can confide a secret. The
physician agrees. The patient says, “I am hired to kill you.” He pulls out a gun and shoots
the physician. The next morning the patient sends an anonymous letter to the physician's
family with all his cash compensation in order to pay for funeral expenses. This patient is
exhibiting which of the following defense mechanisms?

A. Passive aggression
B. Intellectualization
C. Altruism
D. Acting out
E. Dissociation
F. Reaction formation
225. A 57-year-old woman is brought to the emergency room unresponsive. She consumed a
large amount of improperly cooked puffer fish at a Japanese restaurant 30 minutes ago.
Tetrodotoxin is determined to be the cause of poisoning. This toxin binds to and blocks fast
voltage-gated sodium channels in nerve tissue. This patient is most likely suffering from
which of the following neurologic conditions?

A. Sedation
B. Suffocation
C. Indigestion
D. Locked-in syndrome
E. Hemorrhagic infarct from sudden vasoconstriction

226. Sometimes when a neoplasm arises, it is destroyed by natural killer cells or cytotoxic T
lymphocytes before or during metastasis. This process is best described by which
phenomenon?

A. Immune privilege
B. Immune surveillance
C. Immune suppression
D. Tumor suppressor gene activation
E. Autoimmune disease

227. A 30-year-old man is brought to the physician because of a two-hour history of confusion.
His past medical history is significant for a splenectomy performed three years ago due to a
ruptured spleen from an automobile accident. The patient is oriented to the person, but not
place or time. His temperature is 39℃ (102.2℉), blood pressure is 95/55 mm Hg, pulse is
123/min, and respirations are 32/min. Physical examination shows profuse sweating. A
blood smear of the patient stained with hematoxylin and eosin (H&E) is shown below.

​Courtesy of ​Paulo Henrique Orlandi Mourao​ and M


​ ikael Häggström

The black arrow is pointing to an inclusion body that most likely contains what material?

A. Deoxyribonucleic acid (DNA)


B. Ribonucleic acid (RNA)
C. Hemoglobin
D. Iron
E. Bacteria

228. A five-year-old boy is brought to the physician because of cuts and bruises obtained by
riding his bicycle into a thornbush. The boy has been living in an orphanage for the past six
months since the death of his mother. In this time period, he has been brought to the office
20 times due to activities that resulted in physical injury. After the appropriate treatment is
given, the boy says, “I always look forward to seeing you. You are the only person who
cares about me after my mom left.” Which response by the physician is most appropriate to
make to this boy?

A. “I am sorry for your loss, but I am your physician, not your mother.”
B. “I am glad I can help. It’s my job.”
C. “Tell me more about your mom.”
D. “I always look forward to seeing you too, but you should be more careful in the future.”
E. “Do you understand that death is permanent and that your mom is never coming back?”

229. A 19-year-old man comes to the physician for an annual health exam. He has no
complaints. The patient has a history of angina pectoris but takes no medications. His
mother and father have familial hypercholesterolemia. He has a one pack per day smoking
history for the past four years, drinks two bottles of vodka per day, and has recently started
using crack cocaine recreationally. His blood pressure is 146/95 mm Hg, pulse is 93/min,
and BMI is 40 kg/m​2​. Physical examination shows xanthelasma and corneal arcus.
Laboratory results reveal the following.

Blood:
Cholesterol total: 720 mg/dL
Fasting glucose: 123 mg/dL
Hemoglobin A​1C​: 6.3%

What possible health risk is of immediate concern for this patient?

A. Liver cirrhosis
B. Diabetes mellitus
C. Cancer
D. Myocardial infarction
E. Sleep apnea

230. A 22-year-old primigravida woman gives birth to a healthy baby girl through an
uncomplicated vaginal delivery. The woman has multiple sexual partners and is uncertain
who the biological father is. The woman’s boyfriend agrees to a paternity test. Cheek swabs
are taken from the woman, her boyfriend, and neonate for DNA analysis. Polymerase chain
reaction (PCR) results show that half the neonate’s DNA is identical to half of the woman's
DNA. The other half of the neonate’s DNA is identical to half of the boyfriend’s DNA. What
biological principle accounts for these laboratory findings?

A. Chargaff’s rules
B. Linkage disequilibrium
C. Chromothripsis
D. Law of segregation
E. Compound heterozygosity

231. A 26-year-old man visits his physician for a cardiac stress test. The patient’s starting pulse
is 95/min. After 30 minutes of strenuous exercise, his pulse increases to 100/min. His pulse
gradually drops back down to 95/min at rest. Which of the following conditions best
represents the results of this patient’s cardiac stress test?

A. Rheumatic heart disease


B. Aortic stenosis
C. Hypertrophic cardiomyopathy
D. Heart transplant
E. Sedentary lifestyle

232. A 21-year-old woman comes to office due to concerns over heart health. She is currently
healthy but has a family history of cardiovascular disease. A social worker hands the
patient a questionnaire. The questionnaire includes asking the patient about her
demographics, lifestyle, medical history, and family history, as well as specific factors
related to her heart, such as smoking and blood pressure. After submitting the
questionnaire, the physician informs the patient that her cardiovascular age is 30. This
questionnaire is an example of which of the following healthcare tools?

A. Cross-sectional study
B. Health risk assessment
C. Primary prevention
D. Root-cause analysis
E. Cognitive-behavior therapy
F. Community-level intervention

233. A 15-year-old boy comes to the physician due to severe itching one hour after a school
picnic. The patient ate a peanut butter and jelly sandwich for lunch, a food choice he only
recalls eating twice in his life. His past medical history is unremarkable. An image of the
patient’s right forearm is shown.
​Courtesy of James Heilman, MD

Metabolism of which of the following amino acids is necessary for the development of this
patient’s skin lesions?

A. Alanine
B. Tyrosine
C. Tryptophan
D. Histidine
E. Glutamine
F. Phenylalanine

234. A physician becomes addicted to morphine for pain after being injured in war. He decides
to invent a new liquid solution that will replace his addiction. This new liquid solution most
likely contains which of the following drugs?

A. Atropine
B. Bethanechol
C. Cocaine
D. Dopamine
E. Epinephrine

235. A 31-year-old man comes to the physician because of a 10-day history of lower back pain.
He works as a video game designer and sits at his computer for long periods of time. The
patient has had a Trendelenburg gait since the age of eight due to complications from
Legg–Calvé–Perthes disease. Physical examination shows kyphosis. Localized tenderness
is felt over the wings of the ilium. Extension of the back exacerbates the pain. This patient
most likely overused which of the following muscles?

A. Serratus anterior
B. Psoas major
C. Quadratus lumborum
D. Gluteus medius
E. Semitendinosus

236. Human immunodeficiency virus (HIV) can infect monocytes, macrophages, dendritic cells,
T​h​ cells, and T​reg​ cells because of which shared characteristic of these cells?
A. They are white blood cells
B. They participate in phagocytosis
C. They originate in the bone marrow
D. They are CD4+
E. They are nucleated

237. Humans have diverse physical characteristics such as eye color, skin color, and hair color.
These characteristics are coded by different genes on different loci. However, when two
recessive alleles of OCA1 are inherited, the resulting individual will have blue eyes, fair
skin, and blonde hair regardless what alleles are present on the other loci. OCA1 is an
example of which of the following genetic phenomena?

A. Imprinting
B. Pleiotropy
C. Epistasis
D. Locus heterogeneity
E. Double heterozygosity

238. A 20-year-old woman comes to the physician for an annual physical as required by the
International Powerlifting Federation (IPF). She is a competitive class I powerlifter. Her
height is 155 cm (5 ft,1 in), weight 84 kg (185 lbs), and BMI 35 kg/m​2​. Her blood pressure is
115/70 mm Hg, and pulse is 65/min. Physical examination shows large muscle mass. What
are the expected physiological changes to this patient's vitals during a sustained overhead
barbell lift for three seconds?

Systolic blood Diastolic blood Pulse pressure Myocardial


pressure pressure contractility

A. ↓ ↓ ↔ ↔

B. ↔ ↑ ↓ ↔

C. ↑ ↔ ↑ ↔

D. ↑ ↑ ↔ ↔

E. ↑ ↑ ↑ ↑

239. A patient with diabetes mellitus is protected against which of the following cardiovascular
conditions compared to those without diabetes mellitus?

A. ​Myocardial infarction​ due to increased myocardial use of serum glucose for energy
B. ​Atherosclerosis​ due to increased lipid clearance of vessel walls by macrophages
C. ​Arteriosclerosis​ due to forced vasodilation from glucose concentrated viscous blood
D. ​Abdominal aortic aneurysm​ due to glycosylation of matrix proteins in the aortic wall

240. A 14-year-old girl is brought to the emergency department unresponsive after a motor
vehicle collision. Her blood pressure is 70/40 mm Hg and pulse is 145/min. Physical
examination shows pallor. A bracelet found on the patient’s left wrist is shown in the image
below.

What is the most appropriate treatment for this patient?

A. Standard O negative blood


B. O negative blood washed of residual plasma
C. O negative blood with IgA supplementation
D. O negative blood but without resuscitation if indicated (eg, CPR)
E. 0.9% saline without blood

241. A 39-year-old man walks into the emergency department complaining of thirst, fatigue, and
lightheadedness. He is a gambler who had won a large sum of money at the casino the
previous night. He was then mugged by a group of prostitutes and stranded in the Mojave
Desert where he has been wandering about for the past 16 hours. The patient is oriented to
person and place, but not time. His blood pressure is 100/60 mm Hg and pulse is 90/min.
Physical examination shows pallor and diaphoresis. Compared to the day before, this
patient is expected to have experienced which of the following physiologic changes?

Renin Aldosterone Vasopressin Urine


(ADH) specific
gravity

A. ↑ ↑ ↑ ↑

B. ↑ ↓ ↑ ↔

C. ↓ ↑ ↔ ↔

D. ↓ ↓ ↔ ↓

E. ↓ ↓ ↓ ↓

242. The patient above is observed to excrete dark yellow urine. This urine color most likely
indicates a high concentration of which of the following substances?
A. Homogentisic acid
B. Porphobilinogen
C. Uric acid
D. Urea
E. Urobilín (Urochrome)

243. A physician who is a breast cancer specialist has 10 female patients who are confirmed to
have the breast cancer type 1 (BRCA1) gene mutation. None of these patients currently
have breast cancer. Past medical research has shown that approximately 72% of women
who carry the BRCA1 gene mutation will develop breast cancer in their lifetime. What is the
probability that at least one of these 10 female patients will not develop breast cancer?

A. 0.28 x 10
B. 0.72 x 10
C. 0.28​10
D. 0.72​10
E. 1 - 0.28​10
F. 1 - 0.72​10

244. A new screening test is created to detect the presence of Wolf-Hirschhorn syndrome in
neonates, a rare genetic disease that has a prevalence of 1/50,000 (low prevalence). The
screening test has a sensitivity and specificity of 90% each. This screening test consists of
a checklist of common physical features seen in patients with this disease. If all features on
the checklist are positive, then the screening test is positive. If a neonate receives a
positive screening result, a diagnostic cytogenetic test will be performed to confirm the
diagnosis. A neonate screens positive for Wolf-Hirschhorn syndrome. Cytogenetic analysis
is most likely to reveal that this neonate falls into which of the following categories?

A. True positive
B. True negative
C. False positive
D. False negative

245. Succinylcholine is the murder weapon of choice for healthcare criminals. A high dose of
this drug will paralyze nearly all muscles in the human body within seconds thereby
inducing suffocation. What makes this weapon a favorite choice is the fact that catabolism
of this drug is equally as fast. Enzymes break down succinylcholine into succinic acid and
choline, two substances that are naturally abundant in the body. For this reason, it is
difficult to find evidence succinylcholine was ever used, nor that a murder ever took place.
In order for succinylcholine to be fully metabolized within seconds of administration, the
enzyme responsible for this action is most likely located in which of the following
compartments of the human body?

A. Lungs
B. Endocardium
C. Liver
D. Kidneys
E. Plasma

246. A 51-year-old man is brought to the physician with dyspnea and agonal breathing. Shortly
after arrival, he collapses and becomes unresponsive. Medical records show that he is
diagnosed with Romano-Ward syndrome but currently takes no medications. An
electrocardiogram reveals ventricular fibrillation. Cardiopulmonary resuscitation (CPR) and
defibrillation are subsequently begun and the patient is revived. Mexiletine is then
prescribed to the patient for out-patient care. This drug is protective against ventricular
fibrillation by producing which of the following effects on the heart?

A. Slows action potential conduction velocity


B. Shortens action potential duration
C. Slows sinus node discharge rate
D. Slows atrioventricular nodal conduction velocity
E. Prolongs refractory period of cardiac myocytes

247. A 23-year-old man comes to the physician due to a 12-month history of painful lesions in
his left axilla. He explains that the lesions often produce pus and have a foul smelling odor;
he has started to use excess deodorant and wear long sleeve shirts as a consequence.
The patient currently takes no medications and his family history is unremarkable. He does
not use tobacco, alcohol, or illicit drugs. An image of the patient’s left axilla is shown below.

Oral doxycycline is prescribed, and the patient's symptoms resolve over the ensuing
weeks. What is the most likely diagnosis?

A. Acanthosis nigricans
B. Hidradenitis suppurativa
C. Pityriasis rosea
D. Acne vulgaris
E. ​Mycosis fungoides
248. A 21-year-old man comes to the physician because of three distinct lesions on his glans
penis that cause discomfort during intercourse. He is sexually active with four current
partners but uses condoms infrequently. Biopsy of the lesions confirm genital warts. Topical
imiquimod is prescribed for direct application onto the lesions. This medication helps treat
genital warts through which of the following mechanisms of action?

A. Direct cytotoxicity
B. Prevention of lysogenic conversion
C. Augmentation of the immune response
D. Inhibition of viral DNA dependent RNA polymerase
E. Ubiquitination of viral E6 and E7 proteins for proteasome degradation

249. A 33-year-old woman is brought to the emergency department after a car collided with her
driver's side door on the left. She is hypotensive and unresponsive. Physical examination
and imaging reveals a massive hemorrhage involving the descending colon. After
preventing further bleeding, the surgeon resects the inferior mesenteric artery and a left
hemicolectomy is performed. The proximal two-thirds of the transverse colon is then
sutured to the sigmoid colon. Which of the following arteries is the most direct blood supply
to the sigmoid colon after this patient’s resection?

A. Marginal artery of Drummond


B. Mesenteric meandering artery (arc of Riolan)
C. ​Bühler's anastomotic artery (arc of Bühler)
D. Superior mesenteric artery
E. Internal pudendal artery
F. Inferior pancreaticoduodenal artery

250. A 33-year-old woman is brought to the emergency department after a car collided with her
driver's side door on the left. She is hypotensive and unresponsive. Physical examination
and imaging reveals a massive hemorrhage involving the descending colon. After
preventing further bleeding, the surgeon resects the inferior mesenteric artery and a left
hemicolectomy is performed. The proximal two-thirds of the transverse colon is then
sutured to the sigmoid colon. Which of the following branches of the abdominal aorta is the
primary blood supply to the sigmoid colon after this patient’s resection?

A. Celiac trunk
B. Superior mesenteric artery
C. Renal arteries
D. Gonadal arteries
E. Lumbar arteries

251. An 18-year-old man is brought to the emergency department complaining of left sided
abdominal pain and shoulder pain. He was hit with a baseball bat 30 minutes ago during
high school baseball practice. His blood pressure is 100/60 and pulse is 120/min. Physical
examination shows a large area of discoloration in the upper left abdominal quadrant that is
tender to palpation. An abdominal X-ray reveals fractures of the 10th and 11th ribs on the
left. Damage to which of the following organs is the most likely cause of this patient’s
abnormal vitals?

A. Left lung
B. Heart
C. Liver
D. Spleen
E. Left kidney
F. Colon at the splenic flexure

252. A 20-year-old primigravida woman at 35-weeks gestation is induced into labor after
ultrasound monitoring revealed restricted intrauterine growth. The woman has a history of
epilepsy well controlled with phenytoin. When she found out she was pregnant at 10-weeks
gestation, she decided to cease her use of medications and joined an ​Alcoholics
Anonymous (AA) ​program to limit her consumption of alcohol. She continues to use
unspecified illicit drugs. Physical examination of the neonate shows a bilateral complete
cleft lip, microcephaly, and fingernail hypoplasias. A holosystolic murmur along the lower
left sternal border can be heard on auscultation. What is the most likely diagnosis for this
neonate?

A. Fetal hydantoin syndrome


B. Fetal alcohol syndrome
C. Down syndrome
D. Congenital rubella syndrome
E. Ebstein anomaly

253. A 66-year-old man is brought to the emergency department after being found collapsed in
his garage with his car engine on. A suicide attempt is suspected. The patient has a history
of myasthenia gravis treated with neostigmine, but has not taken his medication recently
due to a feeling of hopelessness complicated by major depressive disorder. The patient is
somnolent, and only oriented to the person, but not time or place. CO-oximeter indicates a
blood carboxyhemoglobin to hemoglobin ratio of 15%. Endotracheal intubation with 100%
oxygen is scheduled. During intubation, succinylcholine is usually the drug of choice for
anesthesia. Succinylcholine is contraindicated in this patient due to which of the following
preexisting conditions?

A. Age
B. Suicide attempt
C. Myasthenia gravis
D. Use of neostigmine
E. Major depressive disorder
F. High blood carboxyhemoglobin level
254. A 14-year-old Asian-American girl is brought to the physician because of a two-week
history of abdominal cramps and watery diarrhea. Her symptoms arise after consuming
large quantities of milk and cheese which she has enjoyed in the past without
complications. The patient’s family history is significant for a one-year-old younger sister
who has been diagnosed with congenital lactase deficiency, a rare autosomal recessive
condition that presents with lactose intolerance from birth. Physical examination of the
patient shows no abnormalities and laboratory tests are unremarkable. This patient’s sister,
and herself, are likely to have which of the following categories of lactose intolerance
respectively?

A. Primary lactase deficiency; primary lactase deficiency


B. Primary lactase deficiency; secondary lactase deficiency
C. Secondary lactase deficiency; primary lactase deficiency
D. Secondary lactase deficiency; secondary lactase deficiency

255. A 79-year-old man comes to the physician with a 12-month history of fatigue and body
aches that has progressively worsened. He has a loss of appetite accompanied by a 6.8 kg
(15 lbs) weight loss in the past year. Past medical history is significant for bacterial
pneumonia treated with antibiotics five months ago. His temperature is 37.7 ℃ (99.9 ℉) and
blood pressure is 130/80 mm Hg. Physical examination shows a large bruise that is visible
on the medial aspect of his right arm. A palpable mass can be appreciated in the upper left
abdominal quadrant just below the left costal margin. Biopsy of the patient’s bone marrow
reveals grade 3 fibrosis. What additional pathologic process is most likely present in this
patient?

A. Splenic atrophy
B. Thrombocythemia
C. Blast crisis
D. Extramedullary hematopoiesis
E. Polycythemia vera

256. A local community has an annual syphilis incidence and prevalence of 8/1000 and 48/1000
respectively. The recent emergence of ​Treponema pallidum ​resistance to penicillin has led
to an increase in duration of disease by three years. If the incidence does not change, all
infected individuals survive, and no new treatment is developed, what would be the
prevalence of this disease 20 years after it reaches a steady state?

A. Unchanged
B. 24/1000
C. 40/1000
D. 54/1000
E. 72/1000
257. A 36-year-old-man comes to the office after being advised to see the physician by his boss
due to poor work productivity. For the past eight months he has often been late to work and
makes excessive phone calls to his daughter’s daycare asking where she is and what she
is doing. The daycare employees report that the patient has trouble departing with his
daughter in the morning, and that he is frequently the first parent to drive in for pickup in the
afternoon. The patient does not report any delusions, hallucinations, disorganized speech,
or physical distress. He does not take any medications or illicit drugs. Past family history is
significant for the loss of his wife and son in a motor vehicle accident one year ago. What is
the most appropriate diagnosis for this patient?

A. Panic disorder
B. Acute stress disorder
C. Post-traumatic stress disorder
D. General anxiety disorder
E. Separation anxiety disorder
F. Brief psychotic disorder
G. Schizophreniform disorder

258. A biopsy of the liver is taken from a cadaver, stained, and observed under a light
microscope as shown below.

The triangle in the image best represents which of the following structures?

A. Hepatic acinus
B. Hepatic vein
C. Hepatic lobule
D. Portal vein
E. Portal lobule

259. An 18-year-old man comes to the physician with a history of fingernail and toenail
dystrophy since birth. He has no other complaints. Physical examination shows thick
wrinkled skin that appears to grow over the nail plate similar to the image shown below.
​Haymarket media

A skin biopsy examined under light microscopy leads to suspicion of dyskeratosis


congenita. Polymerase chain reaction (PCR) from a saliva sample confirms this diagnosis.
Abnormal keratinization of which layer of the epidermis most likely led to the consideration
of dyskeratosis congenita as a diagnosis for this patient?

A. Stratum corneum
B. Stratum lucidum
C. Stratum granulosum
D. Stratum spinosum
E. Stratum basale

260. Veins deliver nutrients to the body. Which of the following veins are the exception?

A. Systemic veins
B. Pulmonary veins
C. Portal vein
D. Fetal veins
E. Plant veins

261. Most systemic veins have valves that prevent backflow of blood. Based on this information,
which of the following veins do not have valves?

A. Brachial veins
B. Jugular veins
C. Femoral veins
D. Saphenous veins
E. Inferior vena cava

262. A 43-year-old woman comes to the physician with a 11-day history of productive cough,
dyspnea, and chest pain. She is a recent immigrant from Vietnam and does not have an
adequate medical record. Her temperature is 38 ℃ (100.5℉). Throat examination indicates
cervical lymphadenopathy. Flow cytometry reveals a CD4 T-lymphocyte count of 190
cells/mm​3​ and abdominal X-ray shows hepatosplenomegaly. Two sputum samples are
obtained and plated on separate Sabouraud agars. The sample cultured at room
temperature grows mold, but the incubated sample grows yeast. Which of the following
organisms is the most likely cause of this patient’s symptoms?

A. ​Staphylococcus aureus
B. ​Mycobacterium tuberculosis
C. ​Candida albicans
D. ​Aspergillus fumigatus
E. ​Talaromyces marneffei​ (formerly ​Penicillium marneffei)​

263. A 28-year-old man comes to the physician for an annual checkup. He has no complaints.
Past family history is significant for his father dying of small cell lung cancer at the age of
57. The patient has smoked half a pack of cigarettes per day for the past 10 years, drinks
25 shots of vodka every weekend since college, and “swears like a truck driver.” He often
must install polyvinyl chloride pipes as a plumber. His diet mainly consists of grains and
nuts such as sesame seeds, sunflower seeds, and peanuts. Vital signs are within the
normal range and physical examination is unremarkable. After the exam, the patient says,
“My activities are almost identical to my father’s, but I do not want to die of lung cancer like
him.” The physician should advise the patient to cease which of the following activities?

A. Smoking cigarettes
B. Drinking vodka
C. “Swearing like a truck driver”
D. Installing polyvinyl chloride pipes
E. Eating stored grains and nuts

264. A five-year-old girl is brought to the physician because of frequent nosebleeds and easy
bruising. The child’s parents are second cousins and first generation immigrants from Iran.
Physical examination shows a large bruise on the left knee. Plasma analysis indicates
normal prothrombin time (PT), activated partial thromboplastin time (aPTT), platelet count,
and bleeding time. A urea lysis test suggests defective cross-linking of fibrin. X-ray of the
patient's left knee reveals hemarthrosis. What is the most likely diagnosis?

A. Hemophilia A
B. Hemophilia C
C. Factor XIII deficiency
D. Von Willebrand disease
E. Immune thrombocytopenia
F. Protein C deficiency

265. A 45-year-old woman, gravida 3, para 2, at 26-weeks gestation comes to the physician for
routine prenatal care. The patient is concerned that her and her husband’s advanced age
puts their fetus at risk for birth defects. Her husband is 67-years-old. The patient consumes
400 µg of folate daily but otherwise takes no medications. Family history is unremarkable.
She does not use tobacco, alcohol, or illicit drugs. Ultrasonography indicates fetal
abnormalities that warrant amniocentesis. Molecular genetic testing of the amniotic fluid
reveals a fibroblast growth factor receptor 3 (FGFR3) gene mutation. The fetus will most
likely be born with which of the following abnormalities?

A. Spina bifida
B. Low birth weight
C. Rhizomelia
D. Ventricular septal defect
E. Smooth philtrum with thin vermilion border

266. A medical school professor is teaching anatomy to first year medical students. He shows
the class the base of a cranium and points to the foramen shown in the image below.

He explains that the internal carotid artery passes along the superior surface of this
foramen, but the foramen itself is mostly occluded with cartilage and fibrous tissue. Which
of the following structures most likely passes through this foramen in a living human being?

A. Maxillary nerve (CN V​2​)


B. Mandibular nerve (CN V​3​) and lesser petrosal nerve
C. Middle meningeal artery and vein
D. Meningeal branch of ascending pharyngeal artery and emissary veins
E. Posterior meningeal artery

267. The classical complement pathway plays an important role in immunity by causing cell
membrane damage. In order for the classical cascade to be activated, the C1 complement
component must bind to specific sites on two different antibody molecules. Based on the
given information, and your outside knowledge, which of the following immunoglobulin
isotypes will be the most efficient at activating the classical complement pathway?

A. IgA dimer
B. IgG​2
C. IgM monomer (B cell receptor)
D. IgM pentamer (secreted by B cells)
E. Cross-linked IgE

268. A 34-year-old man comes to the physician due to chronic malaise. The patient is an
intravenous heroin addict who was diagnosed with acute hepatitis B infection five years
ago. He currently takes no medications. Physical examination shows yellowing of the skin
and sclera. An abdominal X-ray reveals liver cirrhosis. Serum ELISA tests demonstrate the
presence of two different sets of antibodies that bind different antigens. These two sets of
antibodies most likely exhibit which of the following characteristics in relation to each other?

​Characteristics
Isotype Idiotype Allotype

A. Different Same Same

B. Same Different Same

C. Same Different Different

D. Same Same Different

E. Different Different Same

269. A 46-year-old man is brought to the physician due to an eight-day history of confusion
complicated by paracusia. He is a recent immigrant from Asia and has lived off a corn rich
diet. Recent medical history indicates alcohol abuse accompanied by a few episodes of
diarrhea. His temperature is 37 ℃ (98.6 ℉), blood pressure is 100/65 mm Hg, and pulse is
97/min. Physical examination shows thick, hyperpigmented skin on the forearms and
around the neck base. If a biopsy of this patient’s intestinal epithelium is examined, it will
most likely show which of the following features?

A. Atrophy with occasional ulceration


B. Neutrophilic infiltration of the crypt of Lieberkühn
C. Transmural inflammation with noncaseating granulomas
D. Foamy macrophages in the lamina propria
E. Presence of oocysts with modified acid-fast staining

270. A 23-year-old woman volunteers for a pulmonary function test for research data collection.
She is a Boston Marathon athlete with a daily routine that involves jogging 8 miles (13 km)
in the morning followed by 2 hours of resistance training in the afternoon. Her other
activities include biking and swimming. She does not use tobacco, alcohol, or illicit drugs.
Compared to the average person, this woman’s pulmonary function test is likely to show
which of the following changes?
Total Lung Forced Residual
Capacity Vital Volume
Capacity

A. ↑ ↑ ↔

B. ↑ ↓ ↑

C. ↔ ↔ ↔

D. ↓ ↓ ↑

E. ↓ ↓ ↓

271. A couple visits the genetic counselor for preconception counseling. They both have
hemophilia. The woman has two X chromosomes with identical gene defects coding for the
production of factor VIII. The man has a single X chromosome with a gene defect that
codes for factor IX. They have a teenage daughter who does not have hemophilia. DNA
testing of their daughter shows that she has one X with a factor VIII gene defect and the
other X with a factor IX gene defect. Which genetic phenomenon best describes why this
daughter does not have hemophilia?

A. X-inactivation (Lyonization)
B. Complementation
C. Random assortment
D. Epistasis
E. Crossover

272. A 29-year-old man comes to the physician at the urging of his boss due to his decreasing
work productivity over the past six months. The patient is a financial advisor, but he has lost
clients due to his recent lack of concentration over their long term financial goals. He is
unable to give any definitive advice to his clients. He has been using his work computer to
watch online videos related to death late into the night, which he thinks is contributing to his
sleep disturbances and low energy the next morning. The patient reports that he has been
eating excessively and has gained 10 kg (22 lbs) over the last five months. Activities such
as chess and jogging no longer interest him. Past medical history is unremarkable and he
does not use tobacco alcohol or illicit drugs. He denies having episodes of depressed or
elevated mood during this time period nor thoughts of suicide. What is the most appropriate
diagnosis for this patient?

A. Major depressive disorder


B. Persistent depressive disorder (dysthymia)
C. Depression with atypical features
D. Bipolar disorder
E. Adjustment disorder
273. A two-day-old male infant is brought to the physician due to a swelling on the left side of
his head. Delivery was complicated by an inadequately dilated cervix that required the use
of forceps during the second stage of labor. The infant does not appear to be in distress,
but a non-fluctuant swelling over the parietal bone is observed. The swelling does not cross
suture lines. What is the most likely diagnosis for this infant?

A. Subarachnoid hemorrhage
B. Subdural hematoma
C. Epidural hematoma
D. Cephalohematoma
E. Subgaleal hemorrhage
F. Caput succedaneum

274. A 10-year-old girl is brought to the physician due to a seven-day history of movement
problems. She had a mild respiratory infection prior to the onset of symptoms. Her past
medical history is significant for diarrhea, vomiting, and dysphagia in infancy. Physical
examination shows ataxia and nystagmus. Blood samples reveal excess serum lactate.
Polymerase chain reaction (PCR) analysis confirms French Canadian Leigh syndrome, an
autosomal recessive disease caused by cytochrome C oxidase deficiency. A defect in
which of the following organelles is likely responsible for this patient’s presentation?

A. Rough endoplasmic reticulum


B. Golgi apparatus
C. Mitochondria
D. Lysosome
E. Peroxisome

275. A two-day-old male infant is brought to the physician after a seizure event. The delivery
was uncomplicated, but shortly after discharge the infant started to exhibit signs of
irritability, lethargy, and poor feeding. Blood and urine tests show the following results as
compared to a healthy infant.

Ammonium ↑

Blood glutamine ↑

Blood urea nitrogen (BUN) ↓

Urine orotic acid Present

A head CT scan reveals cerebral edema. The deficient enzyme causing this infant’s
symptoms combines carbamoyl phosphate and ornithine to form which of the following
amino acids?

A. L-alanine
B. L-glutamate
C. L-citrulline
D. L-aspartate
E. L-arginine

276. A 47-year-old woman gravida 3, para 0, abortus 3, visits the genetic counselor for
preconception counseling. She has a history of infertility, but her husband is a healthy 6
feet tall (183 cm), 30-year-old man with two children from a previous marriage. The woman
takes no medications and does not use tobacco, alcohol, or illicit drugs. If this woman is to
get pregnant, her pregnancy is at increased risk for which of the following complications?

A. Fetal achondroplasia
B. Fetal alcohol syndrome
C. Fetal hydantoin syndrome
D. Fetal gastroschisis
E. Hydatidiform mole

277. A researcher is studying the effects of a peculiar phospholipid on laboratory mice. This
phospholipid is released from the endothelium, immune cells, and platelets. It acts as a
cytokine and is involved in many aspects of general immunity including endothelium
attachment, phagocytosis, and degranulation. High levels of this substance can cause
asthma through bronchoconstriction, and shock via vasodilation. What is the most likely
identity of this phospholipid?

A. Interleukin-6
B. Immunoglobulin epsilon (IgE)
C. Endothelin
D. Platelet-activating factor
E. Leukotriene B​4

278. A 33-year-old primigravida woman at 16 weeks gestation comes to the physician for
routine prenatal care. The patient has a history of syncope triggered by intense emotional
events or by prolonged standing. Her temperature is 37 ℃ (98.6 ℉), blood pressure is
117/75 mm Hg, and pulse is 65/min. She takes no medications and her pregnancy appears
unremarkable. During a blood draw, the patient experiences a brief syncopal episode. Vital
measurements after the episode shows a decrease in blood pressure when standing, and a
decrease in heart rate. Fasting blood glucose is 95 mg/dL. Which of the following
pathological processes most likely explains this patient’s syncope?

A. Diabetes induced autonomic dysfunction


B. Inferior vena cava compression by gravid uterus
C. Vasodilation due to septic shock
D. Acute volume depletion from the blood draw.
E. Withdrawal of sympathetic activity and enhancement of parasympathetic activity
279. A 40-year-old man with familial hypercholesterolemia comes to the physician for
cholesterol monitoring. His medications include atorvastatin, warfarin, aspirin, metoprolol,
and captopril. Past medical history is significant for a myocardial infarction three years ago.
A lipid panel reveals a blood LDL cholesterol level of 240 mg/dL. The physician prescribes
cholestyramine, a bile acid sequestrant, to attempt to lower the cholesterol further. The
patient accepts this medication. Prior to discharge, the physician should consider
recommending which of the following additional actions for this patient?

A. Increasing the dose of atorvastatin


B. Removing atorvastatin
C. Replacing metoprolol with propranolol
D. Administering tissue plasminogen activator (tPA)
E. Strict vegan diet

280. An 11-year-old boy is brought to the emergency department with extreme pain in his right
lateral hip and thigh. He was the passenger in a motor vehicle collision 15 minutes ago.
The patient is unable to move his right leg, but it is unclear if this inability is due to pain or
nerve damage. X-ray reveals a posterior hip dislocation that involves severance of the
ligamentum teres as illustrated in the image below.

​Courtesy of Mike Walden

If the severed ligament is not properly repaired, this patient is most likely to experience
which of the following complications?

A. Sepsis
B. Wet gangrene
C. Anterior compartment syndrome
D. Posterior compartment syndrome
E. Stunted growth spurt

281. A 68-year-old man comes to the physician due to a four week history of mild back and
abdominal discomfort. His past medical history is significant for coronary artery disease and
hypertension. The patient has a 50-pack-year smoking history and drinks alcohol socially.
His BMI is 37 kg/m​2​. Physical examination is unremarkable. Abdominal ultrasound reveals
an aortic aneurysm. Surgery to repair the aneurysm is performed. This patient is at
increased risk of developing which of the following diseases as a result of the repair?

A. Atheroembolic renal disease


B. Diabetes mellitus
C. Prinzmetal angina
D. Granulomatosis with polyangiitis
E. Small-cell lung cancer

282. A 26-year-old man comes to the physician due to a 30 minute history of itching on his
chest. He ate shellfish at a local seafood restaurant prior to the onset of symptoms. The
patient’s left chest wall is shown in the image below.

​Courtesy of James Heilman, MD

The lesions on the patient’s chest are determined to be caused in part by leukocytes in the
dermal interstitium. For this patient, which of the following proteins is primarily involved in
the rolling process of the leukocytes prior to their extravasation into the interstitial space?

A. E-selectin
B. P-selectin
C. CD18
D. Intracellular adhesion molecule-1 (ICAM-1)
E. Platelet endothelial cell adhesion molecule 1 (PECAM-1)

283. Researchers have developed a new screening tool to detect ovarian cancer using the
cancer antigen 125 (CA-125) biomarker. Blood samples from 100 women with ovarian
masses detected on pelvic sonography are tested for CA-125 using murine monoclonal
antibodies. Ovarian biopsies are then obtained to confirm the diagnosis. The results are
shown in the two by two table below.

Ovarian biopsy positive Ovarian biopsy negative


CA-125 positive 9 3

CA-125 negative 1 87

Which of the following calculations represent the fall-out rate?

A. 9/(9+1)
B. 3/(9+3)
C. 3/(87+3)
D. 1/(9+1)
E. 1/(87+1)

284. A 19-year-old woman with anorexia comes to the physician due to a 12 hour history of
nausea and vomiting. She has eaten little for the past several months. The patient
underwent a duodenal resection for duodenal adenocarcinoma a year ago, and has since
taken loperamide regularly to prevent diarrhea. Her family history is significant for diabetes
and hypertension. She smokes 12 packs of cigarettes per year and occasionally uses
cocaine, but does not consume alcohol. Her pulse is 110/min. Physical examination shows
diaphoresis, pallor, and dilated pupils. A blood glucose meter reveals hypoglycemia for
which 50% intravenous dextrose is administered. Shortly after treatment, the patient
develops horizontal nystagmus, bilateral abducens palsy, gait ataxia, and confusion. Which
of the following preexisting factors most likely precipitated this patient’s post-treatment
neurological symptoms?

A. Decreased food consumption and duodenal resection


B. Use of loperamide and tachycardia
C. History of duodenal carcinoma and smoking
D. Cocaine abuse and family history of hypertension
E. Hypoglycemia and family history of diabetes

285. A newborn girl is delivered at 36 weeks gestation via C-section due to fetal abnormalities
detected on ultrasound. Her mother is a refugee from Sudan who had been prescribed
chloramphenicol for meningitis at a refugee camp in Egypt prior to learning that she was
pregnant. The newborn is observed to have a distended abdomen, skin discolorations,
hypotonia, hypotension, and irregular breathing. Apgar score is 4 and 6 at 1 and 5 minutes
respectively. Blood analysis reveals increased lactate. This newborn’s symptoms are most
likely caused by prematurity of which of the following organs?

A. Lungs
B. Heart
C. Liver
D. Kidneys
E. Colon
286. Researchers conduct a phase II clinical trial for a new cholesterol-lowering drug known as
bemopedioic acid. Sixty patients with high cholesterol at a university hospital agree to
participate. The patients are randomized and equally divided into either the treatment group
or control group. The researchers and physicians are blinded to both groups. At the hospital
pharmacy, the pharmacist informs each patient if they are receiving bemopedioic acid or
atorvastatin (control). All participants complete the trial. This study is most susceptible to
which of the following types of biases?

A. Selection bias
B. Recall bias
C. Procedure bias
D. Observer bias
E. Length-time bias

287. A 76-year-old woman is brought to the physician after a three-day history of difficulty
speaking. A script of the conversion between the patient and physician is shown below.

Physician: Now, I want you to say some words after me. Say ‘boy’.
Patient: Boy.
Physician: Home.
Patient: Home.
Physician: Seventy-nine.
Patient: Ninety-seven. No … sevinty-sine … siventy-nice….
Physician: Let’s try another one. Say ‘refrigerator’.
Patient: Frigilator … no? how about … frerigilator … no frigaliterlater … aahh! It’s all mixed up!
Script courtesy of ​ ​Robert H. Brookshire. An Introduction to Neurogenic Communication Disorders, 6e. volume. Mosby Year Book, St.
Louis, 2003.

Damage to which of the following nerve tracts is likely responsible for this patient’s
presentation?

A. Medial longitudinal fasciculus


B. Dorsal longitudinal fasciculus
C. Inferior longitudinal fasciculus
D. Arcuate fasciculus
E. Uncinate fasciculus

288. A 30-year-old man with cystic fibrosis comes to the physician with progressive shortness of
breath and hemoptysis. His past medical history is significant for respiratory infections and
failure to thrive due to malabsorption. The patient takes lumacaftor and ivacaftor
inconsistently. His temperature is 38.3 ℃ (101 ℉). A CT scan of the lungs reveals
bronchiectasis. Sputum samples confirm a bacterial infection. A graph of the most common
bacterial infections in cystic fibrosis patients is shown below.
Statistically, what is the most likely bacteria responsible for this patient’s symptoms?

A. ​Escherichia coli
B. ​Staphylococcus aureus
C. ​Haemophilus influenzae
D. ​Burkholderia cepacia
E. ​Pseudomonas aeruginosa

289. A six-year-old girl with cystic fibrosis is brought to the physician for a well-child
examination. Her past medical history is significant for respiratory infections and failure to
thrive due to malabsorption. A graph of the most common bacterial infections in cystic
fibrosis patients is shown below.

What is the approximate probability that this patient does not currently have any respiratory
infections?

A. 5%
B. 20%
C. 50%
D. 60%
E. 70%
290. Several children from a kindergarten develop fever, vomiting, diarrhea, and abdominal
cramps three days after a class picnic. At the picnic, each child was asked to take and
consume a single fruit from a basket mixed with apples, oranges, peaches, and bananas. A
table of the data is shown below.

Got sick Did not get sick

Ate apple 3 2

Ate orange 4 3

Ate peach 5 5

Ate banana 5 4

Which of the following fruits is likely the primary source of the outbreak?

A. Apple
B. Orange
C. Peach
D. Banana

291. A 34-year-old woman comes to the physician for a follow-up of acute right upper quadrant
abdominal pain. She currently has no complaints. Her medications include hydromorphone
and gemfibrozil. Family history is significant for pancreatitis on her paternal side. The
patient does not use tobacco or alcohol, but occasionally uses the illicit drug fentanyl for
synergistic effects with hydromorphone. A lipid panel shows the following results.

Cholesterol, serum:
Total: 230 mg/dL
HDL: 65 mg/dL
LDL: 110 mg/dL
Triglycerides, serum:
540 mg/dL (↓ from 1034 mg/dL)

In addition to encouraging the patient to continue her adherence to prescribed medications,


the physician should consider recommending the patient increase her consumption of
which of the following items?

A. Fish oil
B. Simple carbohydrates
C. Fat soluble vitamins
D. Lean meat
E. Fentanyl
292. In light of the recent deaths of Alex Trebek, Justice Ruth Bader Ginsburg, and John Lewis,
a researcher petitions Congress for national pancreatic cancer screening. She claims that
screening for pancreatic adenocarcinoma with carbohydrate antigen 19-9 (CA 19-9) will
produce a high value when dividing the probability of a subject with the disease testing
positive by the probability of a subject without the disease testing positive. The researcher
is claiming which of the following statistical measures is high?

A. True positive rate


B. False positive rate
C. Positive predictive value
D. Positive likelihood ratio

293. A 54-year-old man with type I diabetes comes to the physician due to chronic intermittent
claudications and a right foot ulcer. Examination of the foot shows a bloody three
centimeter in diameter ulcer. Blood pressure measurements reveal a low ankle-brachial
index (ABI). Angiography is then performed by inserting a catheter into the common
femoral artery and injecting a radiopaque contrast agent. X-ray of the right lower limb
shows the contrast agent reaching the foot but does not travel through the superficial
femoral artery. The contrast agent traveled through which of the following arteries to reach
the foot?

A. Internal iliac artery


B. Obturator artery
C. Medial circumflex femoral artery
D. Lateral circumflex femoral artery
E. Descending genicular artery

294. A 57-year-old man comes to the physician due to chronic itching in his limbs for the past
several months. The use of various soaps while bathing exacerbates the condition. The
patient’s past medical and family histories are insignificant and he does not use tobacco,
alcohol, or illicit drugs. Physical examination shows generalized xerosis with skin lesions
similar to the image seen below.
Laboratory imaging and blood tests are unrevealing. What is the most likely diagnosis?

A. Herpes zoster (shingles)


B. Psoriasis
C. Nummular eczema
D. Urushiol-induced contact dermatitis
E. Polycythemia vera

295. A 32-year-old man comes to the physician for a tuberculin sensitivity test as required by his
new employer. He informs the physician that he needs the test results urgently to secure
his employment. The physician has recently learned from a family friend that this patient
has been having a clandestine affair with his wife. The physician administers the
appropriate dose intradermally and schedules the patient to return in 72 hours for skin
analysis. Three days later, the patient returns to the office only to be informed by the
secretary that his physician has unexpectedly canceled the appointment and is taking an
early lunch break. The physician is exhibiting which of the following defense mechanisms?

A. Displacement
B. Passive aggression
C. Acting out
D. Reaction formation
E. Altruism

296. A 22-year-old woman comes to the physician due to chronic nasal congestion and
headaches. Her symptoms are worse on the weekends when she hosts house parties with
large quantities of alcohol. She also notes that her congestion worsens when exposed to
outdoor temperatures after long hours indoors. The patient has no known allergies and
does not use nasal decongestants. She does not use tobacco or illicit drugs, but drinks
socially on her weekend gatherings. Intranasal examination reveals mucosal edema with
clear mucus secretions. What is the most likely diagnosis?

A. Nasal vestibulitis
B. Rhinitis medicamentosa
C. Honeymoon rhinitis
D. Gustatory rhinitis
E. Non-air flow rhinitis
F. Vasomotor rhinitis

297. A 71-year-old man comes to the ophthalmologist due to a two-week history of partial vision
loss. Loss of vision first occurred in the left eye then followed by the right eye. Visual field
test reveals binasal hemianopia. What is the most likely cause of this patient’s vision loss?

A. Temporal arteritis
B. Retinal emboli
C. Craniopharyngioma
D. Pituitary adenoma
E. Calcification of the internal carotid arteries

298. An epidemiologist discovers that a recent outbreak of ​Legionella pneumophila​ in a small


​ egionnaires’ disease. In the ensuing year, 15% of
town has led to many cases of​ L
symptomatic individuals died, while the rest recovered. This percentage represents which
of the following epidemiological measurements?

A. Crude mortality rate


B. Cause-specific mortality rate
C. Proportionate mortality rate
D. Case fatality rate
E. Infection fatality rate
F. Attack rate

299. A 10-month-old boy is brought to the physician with a weeklong history of shortness of
breath, fever, non-productive cough, diarrhea, and night sweats. Family history is significant
for a brother with hyper-IgM syndrome. An immunoglobulin blood test shows the following
measurements.

Immunoglobulins, serum
IgA 0 mg/dL
IgE 0 IU/mL
IgG 60 mg/dL
IgM 690 mg/dL

Chest X-ray reveals pneumonia; ​Pneumocystis jirovecii​ is subsequently identified on


bronchoalveolar lavage. Which cluster of differentiation (CD) is most likely defective in this
patient?

A. CD4
B. CD14
C. CD20
D. CD40
E. CD154

300. A 55-year-old man comes to the physician with a two-day history of malaise and productive
cough. The patient’s past medical history is significant for gastroesophageal reflux disease.
His temperature is 37.8 ℃ (100 ℉). A complete blood count reveals leukocytosis and
subsequent chest imaging uncovers pneumonia with pulmonary infiltrates. Which of the
following medications is the most likely cause of this patient’s presentation?

A. Amiodarone
B. Diltiazem
C. Hydroxychloroquine
D. Esomeprazole
E. Sildenafil

301. A 29-year-old primigravida woman undergoes elective chorionic villus sampling (CVS) after
her genetic counselor informs her that her fetus could be at risk for health problems. The
patient’s husband has pseudohypoparathyroidism type 1A. Polymerase chain reaction
(PCR) results show the fetus is heterozygous for the ​GNAS1​ gene mutation. What
physiological or anatomical abnormalities would be expected in this fetus if it were to be
carried to term?

A. No abnormalities
B. Kidney resistance to parathyroid hormone
C. Skeletal defects
D. Elevated serum parathyroid hormone and phosphorus
E. Hypocalcemia

302. A nine-month-old boy is brought to the physician because of failure to thrive and inability to
meet developmental milestones. His past medical history is significant for premature birth.
Physical examination shows hypotonia with brittle hair. A blood sample analysis reveals low
copper levels, and subsequent genetic testing uncovers a gene mutation in ​ATP7A​. Which
additional physical finding could be consistent with this patient’s underlying disease?

A. Skin hyperpigmentation
B. Copper deposits in the corneas
C. Hypopigmented irises
D. Osteopetrosis
E. Psychosis

303. A 24-year-old woman is suspected of having a previous infection with ​Chlamydia


trachomatis​.​ H
​ er serum is placed into a flask and heated high enough to denature her
complements, but not her antibodies. Complement from guinea pig is then added to the
serum. ​Chlamydia e ​ lementary bodies are added following the guinea pig complements.
Finally, antibody-coated sheep erythrocytes are poured into the flask. The erythrocytes
settle at the bottom of the flask after several minutes. A positive test result is reported. This
procedure is an example of which of the following immunological assays?

A. Direct Coombs test


B. Indirect Coombs test
C. Enzyme-linked immunosorbent assay (ELISA)
D. Complement fixation test
E. Latex agglutination assay
304. A 19-year-old woman gives birth at 42 weeks gestation to an infant boy. She had no
prenatal care during her pregnancy. The patient’s past medical history is significant for
acne vulgaris and bipolar disorder well controlled with isotretinoin and lithium respectively.
She has a half pack per day smoking history for the past year and drinks several cans of
beer on the weekends mixed with powdered cocaine since her 16th birthday. The infant is
cyanotic and tachypneic upon delivery. Physical examination shows microcephaly,
micrognathia, depressed nasal bridge, hypertelorism, and no ears. An electrocardiogram
reveals transposition of the great arteries. Which of the following substances is the most
likely primary cause of this infant's presentation?

A. Isotretinoin
B. Lithium
C. Nicotine
D. Alcohol
E. Cocaine

Answer Key:

1. B
2. B
3. D
4. C
5. A
6. E
7. D
8. C
9. A
10. C
11. B
12. A
13. A
14. D
15. C
16. C
17. C
18. D
19. B
20. C
21. A
22. C
23. E
24. B
25. C
26. E
27. B
28. A
29. E
30. C
31. B
32. A
33. D?
34. C
35. B
36. D
37. B
38. B
39. D
40. D
41. D
42. E
43. C?
44. C
45. D
46. Ignore
47. C
48. E
49. B
50. D
51. D
52. C
53. A
54. A
55. B
56. C
57. A
58. E
59. E
60. A
61. C
62. B
63. E
64. E
65. A
66. D
67. B
68. C
69. E
70. A
71. D
72. B
73. D
74. A
75. C
76. B
77. E
78. B
79. D
80. C
81. B
82. C
83. D
84. C
85. E
86. A
87. E
88. D
89. C?
90. D
91. C
92. A
93. A or E?
94. B
95. B
96. C
97. E
98. B
99. C?
100. B
101. D
102. E
103. C
104. C
105. E
106. C
107. B
108. D
109. B
110. D
111. A
112. B
113. A
114. H
115. C
116. E
117. C
118. D
119. A
120. A
121. C
122. B
123. A
124. C
125. A
126. D
127. C
128. D
129. C
130. A
131. A
132. D
133. D
134. D
135. B
136. A
137. B
138. D
139. E
140. C
141. E
142. G
143. D
144. C
145. B
146. C
147. D
148. C
149. C
150. E
151. B
152. D
153. B or C
154. C
155. C
156. E
157. A
158. E
159. B
160. C
161. A
162. B
163. E
164. C
165. A
166. C
167. E
168. C
169. B
170. D
171. E
172. C
173. C
174. E
175. A
176. D
177. D
178. D
179. B
180. B
181. E
182. A
183. B
184. G
185. D
186. B
187. A
188. C
189. D
190. D
191. A
192. E
193. A
194. B
195. B
196. A
197. A
198. C
199. C
200. D
201. B
202. B
203. D
204. B
205. E
206. C
207. A
208. B
209. C
210. C
211. B
212. C
213. C
214. C
215. A
216. E
217. E
218. D
219. C
220. A
221. B
222. E
223. K
224. C
225. B
226. B
227. A
228. C
229. D
230. D
231. D
232. B
233. D
234. C
235. C
236. D
237. C
238. E
239. D
240. B
241. A
242. E
243. F
244. C
245. E
246. B
247. B
248. C
249. A
250. B
251. D
252. A
253. C
254. A
255. D
256. E
257. E
258. E
259. D
260. A
261. B
262. E
263. A
264. C
265. C
266. D
267. D
268. B
269. A
270. A
271. B
272. A
273. D
274. C
275. C
276. E
277. D
278. E
279. A
280. E
281. A
282. B
283. C
284. A
285. C
286. D
287. D
288. E
289. B
290. A
291. A
292. D
293. D
294. C
295. B
296. F
297. E
298. D
299. E
300. D
301. D
302. C
303. D
304. A

You might also like